You are on page 1of 97

Contents

1 International Mathematics Olympiad 6

1.1 1st IMO, Romania, 1959 . . . . . . . . . . . . . . . . . . . . . . . . . . . . 6

1.2 2nd IMO, Romania, 1960 . . . . . . . . . . . . . . . . . . . . . . . . . . . . 7

1.3 3rd IMO, Hungary, 1961 . . . . . . . . . . . . . . . . . . . . . . . . . . . . 8

1.4 4th IMO, Czechoslovakia, 1962 . . . . . . . . . . . . . . . . . . . . . . . . . 9

1.5 5th IMO, Poland, 1963 . . . . . . . . . . . . . . . . . . . . . . . . . . . . . 10

1.6 6th IMO, USSR, 1964 . . . . . . . . . . . . . . . . . . . . . . . . . . . . . . 11

1.7 7th IMO, West Germany, 1965 . . . . . . . . . . . . . . . . . . . . . . . . . 11

1.8 8th IMO, Bulgaria, 1966 . . . . . . . . . . . . . . . . . . . . . . . . . . . . 12

1.9 9th IMO, Yugoslavia, 1967 . . . . . . . . . . . . . . . . . . . . . . . . . . . 13

1.10 10th IMO, USSR, 1968 . . . . . . . . . . . . . . . . . . . . . . . . . . . . . 14

1.11 11th IMO, Romania, 1969 . . . . . . . . . . . . . . . . . . . . . . . . . . . 16

1.12 12th IMO, Hungary, 1970 . . . . . . . . . . . . . . . . . . . . . . . . . . . . 16

1.13 13th IMO, Czechoslovakia, 1971 . . . . . . . . . . . . . . . . . . . . . . . . 18

1
2 CONTENTS

1.14 14th IMO, USSR, 1972 . . . . . . . . . . . . . . . . . . . . . . . . . . . . . 18

1.15 15th IMO, USSR, 1973 . . . . . . . . . . . . . . . . . . . . . . . . . . . . . 19

1.16 16th IMO, West Germany, 1974 . . . . . . . . . . . . . . . . . . . . . . . . 20

1.17 17th IMO, Bulgaria, 1975 . . . . . . . . . . . . . . . . . . . . . . . . . . . . 21

1.18 18th IMO, Austria, 1976 . . . . . . . . . . . . . . . . . . . . . . . . . . . . 22

1.19 19th IMO, Yugoslavia, 1977 . . . . . . . . . . . . . . . . . . . . . . . . . . 23

1.20 20th IMO, Romania, 1978 . . . . . . . . . . . . . . . . . . . . . . . . . . . 24

1.21 21st IMO, United Kingdom, 1979 . . . . . . . . . . . . . . . . . . . . . . . 25

1.22 22nd IMO, Washington, USA, 1981 . . . . . . . . . . . . . . . . . . . . . . 26

1.23 23rd IMO, Budapest, Hungary, 1982 . . . . . . . . . . . . . . . . . . . . . . 27

1.24 24th IMO, Paris, France, 1983 . . . . . . . . . . . . . . . . . . . . . . . . . 28

1.25 25th IMO, Prague, Czechoslovakia, 1984 . . . . . . . . . . . . . . . . . . . 29

1.26 26th IMO, Helsinki, Finland, 1985 . . . . . . . . . . . . . . . . . . . . . . . 30

1.27 27th IMO, Warsaw, Poland, 1986 . . . . . . . . . . . . . . . . . . . . . . . 31

1.28 28th IMO, Havana, Cuba , 1987 . . . . . . . . . . . . . . . . . . . . . . . . 32

1.29 29th IMO, Camberra, Australia, 1988 . . . . . . . . . . . . . . . . . . . . . 33

1.30 30th IMO, Braunschweig, West Germany, 1989 . . . . . . . . . . . . . . . . 34

1.31 31st IMO, Beijing, People’s Republic of China, 1990 . . . . . . . . . . . . . 36

1.32 32nd IMO, Sigtuna, Sweden, 1991 . . . . . . . . . . . . . . . . . . . . . . . 37

1.33 33rd IMO, Moscow, Russia, 1992 . . . . . . . . . . . . . . . . . . . . . . . . 38


CONTENTS 3

1.34 34th IMO, Istambul, Turkey, 1993 . . . . . . . . . . . . . . . . . . . . . . . 39

1.35 35th IMO, Hong Kong, 1994 . . . . . . . . . . . . . . . . . . . . . . . . . . 40

1.36 36th IMO, Toronto, Canada, 1995 . . . . . . . . . . . . . . . . . . . . . . . 41

1.37 37th IMO, Mumbai, India, 1996 . . . . . . . . . . . . . . . . . . . . . . . . 42

1.38 38th IMO, Mar del Plata, Argentina, 1997 . . . . . . . . . . . . . . . . . . 43

1.39 39th IMO, Taipei, Taiwan, 1998 . . . . . . . . . . . . . . . . . . . . . . . . 44

1.40 40th IMO, Bucharest, Romania, 1999 . . . . . . . . . . . . . . . . . . . . . 45

1.41 41st IMO, Taejon, South Korea, 2000 . . . . . . . . . . . . . . . . . . . . . 46

1.42 42nd IMO, Washington DC, USA, 2001 . . . . . . . . . . . . . . . . . . . . 47

1.43 43rd IMO, Glascow, United Kingdom, 2002 . . . . . . . . . . . . . . . . . . 48

1.44 44th IMO, Tokyo, Japan, 2003 . . . . . . . . . . . . . . . . . . . . . . . . . 49

2 William Lowell Putnam Competition 50

2.1 46th Anual William Lowell Putnam Competition, 1985 . . . . . . . . . . . 50

2.2 47th Anual William Lowell Putnam Competition, 1986 . . . . . . . . . . . 52

2.3 48th Anual William Lowell Putnam Competition, 1987 . . . . . . . . . . . 54

2.4 49th Anual William Lowell Putnam Competition, 1988 . . . . . . . . . . . 56

2.5 50th Anual William Lowell Putnam Competition, 1989 . . . . . . . . . . . 58

2.6 51th Anual William Lowell Putnam Competition, 1990 . . . . . . . . . . . 60

2.7 52th Anual William Lowell Putnam Competition, 1991 . . . . . . . . . . . 62

2.8 53th Anual William Lowell Putnam Competition, 1992 . . . . . . . . . . . 64


4 CONTENTS

2.9 54th Anual William Lowell Putnam Competition, 1993 . . . . . . . . . . . 66

2.10 55th Anual William Lowell Putnam Competition, 1994 . . . . . . . . . . . 68

2.11 56th Anual William Lowell Putnam Competition, 1995 . . . . . . . . . . . 69

2.12 57th Anual William Lowell Putnam Competition, 1996 . . . . . . . . . . . 71

2.13 58th Anual William Lowell Putnam Competition, 1997 . . . . . . . . . . . 73

2.14 59th Anual William Lowell Putnam Competition, 1998 . . . . . . . . . . . 75

2.15 60th Anual William Lowell Putnam Competition, 1999 . . . . . . . . . . . 76

2.16 61st Anual William Lowell Putnam Competition, 2000 . . . . . . . . . . . . 79

2.17 62nd Anual William Lowell Putnam Competition, 2001 . . . . . . . . . . . 80

2.18 63rd Anual William Lowell Putnam Competition, 2002 . . . . . . . . . . . 81

3 Asiatic Pacific Mathematical Olympiads 84

3.1 1st Asiatic Pacific Mathematical Olympiad, 1989 . . . . . . . . . . . . . . . 84

3.2 2nd Asiatic Pacific Mathematical Olympiad, 1990 . . . . . . . . . . . . . . 85

3.3 3rd Asiatic Pacific Mathematical Olympiad, 1991 . . . . . . . . . . . . . . 86

3.4 4th Asiatic Pacific Mathematical Olympiad, 1992 . . . . . . . . . . . . . . . 86

3.5 5th Asiatic Pacific Mathematical Olympiad, 1993 . . . . . . . . . . . . . . . 87

3.6 6th Asiatic Pacific Mathematical Olympiad, 1994 . . . . . . . . . . . . . . . 88

3.7 7th Asiatic Pacific Mathematical Olympiad, 1995 . . . . . . . . . . . . . . . 89

3.8 8th Asiatic Pacific Mathematical Olympiad, 1996 . . . . . . . . . . . . . . . 90

3.9 9th Asiatic Pacific Mathematical Olympiad, 1997 . . . . . . . . . . . . . . . 91


CONTENTS 5

3.10 10th Asiatic Pacific Mathematical Olympiad, 1998 . . . . . . . . . . . . . . 92

3.11 11th Asiatic Pacific Mathematical Olympiad, 1999 . . . . . . . . . . . . . . 93

3.12 12th Asiatic Pacific Mathematical Olympiad, 2000 . . . . . . . . . . . . . . 93

3.13 13th Asiatic Pacific Mathematical Olympiad, 2001 . . . . . . . . . . . . . . 94

3.14 14th Asiatic Pacific Mathematical Olympiad, 2002 . . . . . . . . . . . . . . 95

3.15 15th Asiatic Pacific Mathematical Olympiad, 2003 . . . . . . . . . . . . . . 96


Chapter 1

International Mathematics Olympiad

1.1 1st IMO, Romania, 1959


1. Prove that the fraction
21n + 4
14n + 3
is irreductible for every natural number n

2. For what real values of x is


q √ q √
x + 2x − 1 + x − 2x − 1 = A

given (a) A = 2, (b) A = 1, (c) A = 2, where only non-negative real numbers are
admitted for square roots?

3. Let a, b, c be real numbers. Consider the quadratic equation in cos x: a cos 2 x +


b cos x + c = 0. Using the numbers a, b and c, form a quadratic ecuation in cos 2x,
whose roots are the same as those of the original ecuation. Compare the ecuations
in cos x and cos 2x for a = 4, b = 2 and c = −1

4. Construct a right triangle with hypotenuse c such that the median drawn to the
hypotenuse is the geometric mean of the two legs of the triangle.

5. An arbitrary point M is selected in the interior of the segment AB. The squares
AM CD and M BEF are constructed on the same side of AB, sith the segments AM
and M B as their respective bases. The circles circumscribed abut these squares,

6
1.2. 2N D IMO, ROMANIA, 1960 7

with centers P and Q intersect at M and also at another point N . Let N 0 denote the
intersection of the straight lines AF and BC.

(a) Prove that the points N and N 0 coinside.


(b) Prove that the straight lines M N pass throught a fixed point S independent of
the choice of M .
(c) Find the locus of the midpoints of the the segment P Q as M varies between A
and B.

6. Two planes, P and Q, intersect along the line p. The point A is given in the plane
P , and the point C in the plane Q; neither of these points lies on the straight line p.
Construct an isosceles trapezoid ABCD (with AB parallel to CD) in which a circle
can be inscribed, and with vertices B and D lying in the planes P and Q respectively.

1.2 2nd IMO, Romania, 1960


1. Determine all three-digit numbers N having the property that N is divisible by 11,
N
and 11 is equal to the sum of the squares of the digits of N .

2. For what values of the variable x does the following inequality hold?
4x2
 √ 2 < 2x + 9
1 − 1 + 2x

3. In a given right triangle 4ABC, the hypotenuse BC, of lenght a, is dividen into n
equal parts (n an odd integer). Let α be the acute angle subtending, from A, that
segment which contains the middle point of the hypotenuse. Let h be the lenght of
the altitude to the hypotenuse of the triangle. Prove:
4nh
tan α =
(n2− 1) a

4. Construct a triangle 4ABC, given ha , hb (the altitudes fron A and B) and ma , the
median from vertex A.

5. Consider the cube ABCDA0 B 0 C 0 D 0 (whith face ABCD directly above face A0 B 0 C 0 D 0 ).

(a) Find the locus of the midpoints of segment XY , where X is any point of AC
and Y is any point of B 0 D 0 .
8 CHAPTER 1. INTERNATIONAL MATHEMATICS OLYMPIAD

(b) Find the locus of points Z which lie on the segment XY of part (a) with ZY =
2XZ.
6. Considere a cone of revolution with an inscribed sphere tangent to the base of the
cone. A cylinder is circumscribed about this sphere so that one of its bases lies in
the base of the cone. Let V1 be the volume of the cone and V2 the volumen of the
cilinder.
(a) Prove that V1 6= V2 .
(b) Find the smallest number k for which V1 = kV2 , for this case, construct the
angle subtended by a diameter of the base of the cone at the vertex of the cone.
7. An isosceles trapezoid with bases a and c, and altitude h is given.
(a) On the axis of symmetry of this trapezoid, find all points P such that both legs
of the trapezoid subtended right angles at P .
(b) Calculate the distance of P from either base.
(c) Determine under what conditions such points P actually exist. (Discuss varius
case that might arise)

1.3 3rd IMO, Hungary, 1961


1. Solve the system of equations:

x+y+z = a
x + y 2 + z 2 = b2
2

xy = z 2

where a and b are constants. Give the conditions that a and b must satisfy so that
x, y, z (the solutions of the system) are distinct positive numbers.

2. Let a, b, c the sides of a triangle, and T its area. Prove: a2 + b2 + c2 ≥ 4 3T . In
what case does the equality hold?
3. Solve the equation cosn x − sinn x = 1, where n is a natural number.
4. Consider the triangle 4P1 P2 P3 and a point P within the triangle. Lines P P1 , P P2 ,
P P3 intersect the opposite side in points Q1 , Q2 , Q3 respectively. Prove that, of the
numbers PP1QP1 , PP2QP2 , PP3QP3 at least one is less than or equal to 2 and at least one is
grater than or equal to 2.
1.4. 4T H IMO, CZECHOSLOVAKIA, 1962 9

5. Construct triangle 4ABC if AC = b, AB = c and ^AM B = ω, where M is the


midpoint of the segment BC and ω < 90◦ . Prove that a solution exists and only if
b tan ω2 ≥ c < b. In what case does the equality hold?

6. Considere a plane ε and three non-collinear points A, B, C on the same side of ε;


suppose the plane determined by these three points is not parallel to ε. In plane a take
three arbitrary points A0 , B 0 , C 0 . Let L, M, N be the midpoints of segments AA0 ,
BB 0 , CC 0 ; let G the centroid of triangle 4LM N (We will not considere positions of
A0 , B 0 , C 0 such that the points L, M , N do not form a triangle) What is the locus of
point G as A0 , B 0 , C 0 range independently over the plane ε?

1.4 4th IMO, Czechoslovakia, 1962

1. Find the smallest natural number n which has the following properties:

(a) Its decimal representation has 6 as the last digit.


(b) If the last digit 6 is erased and placed in front of the remaining digits, the
resulting number is four times as large as the original number n

2. Determine all real number x which satisfy the inequality:


√ √ 1
3−x− x+1>
2

3. Consider the cube ABCDA0 B 0 C 0 D 0 (ABCD and A0 B 0 C 0 D 0 are the upper and lower
bases, respectively, and edges AA0 , BB 0 , CC 0 , DD 0 are parallel) The point X moves
at constant speed along the perimeterof the square ABCD in the direction ABCDA,
and the point Y moves at the same rate along the perimeter of the square B 0 C 0 CB
in the direction B 0 C 0 CBB 0 . Points X and Y begin their motion at the same instant
from the starting position A and B 0 , respectively. Determine and draw the locus of
the midpoints of the segment XY .

4. Solve the ecuation cos2 x + cos2 2x + cos2 3x = 1

5. On the circle K there are given three distinct points A, B, C. Construct (using only
straightedge and compasses) a fourth point D on K such that a circle can be inscribed
in the cuadrilateral thus obtained.
10 CHAPTER 1. INTERNATIONAL MATHEMATICS OLYMPIAD

6. Considere an isosceles triangle. let r be the radius of its circumscribed circle and ρ
the radius of its inscribed circle. Prove that the distance d between the centers of
these two circles is q
d = r (r − 2ρ)

7. The tetrahedon SABC has the following propoerty: there exists five spheres, each
tangent to the edges SA, SB, SC, BC, CA, AB or their extentions.

(a) Prove that the tetrahedron SABC is regular.


(b) Prove conversely that for every regular tetrahedron five such spheres exist.

1.5 5th IMO, Poland, 1963


√ 2 √
1. Find all real roots of the equation x − p + 2 x2 − 1 = x, where p is a real param-
eter.

2. Point A and segment BC are given. Determine the locus of points in space which
are vertices of right angles with one side passing throught A, and the other side
intersecting the segment BC.

3. In an n−gon all of whose interior angles are equal, the lenght of consecutive sides
satisfy the relation a1 ≥ a2 ≥ · ≥ an . Prove that a1 = a2 = · = an .

4. Find all solution x1 , x2 , x3 , x4 , x5 of the system

(1) x 5 + x2 = yx1
(2) x 1 + x3 = yx2
(3) x 2 + x4 = yx3
(4) x 3 + x5 = yx4
(5) x 4 + x1 = yx5

where y is a parameter

5. Prove that
π 2π 3π 1
cos − cos + cos =
7 7 7 2
6. Five students, A, B, C, D, E, took part in a contest. One prediction was that
contestants would finish in the order ABCDE. This prediction was very poor. In fact
no contestant finished in the position predicted, and no two contestants predicted to
finish consecutively actually did so. A second prediction has the contestants finishing
1.6. 6T H IMO, USSR, 1964 11

in the order DAECB. This prediction was better. Exactly two of the contestants
finished in the places predicted, and two disjoint pairs of students predicted to finish
consecutively actually did so. Determine the order in which the contestants finished.

1.6 6th IMO, USSR, 1964


1. (a) Find all positive integers n for which 2n − 1 is divisible by 7.
(b) Prove that there is not positive integer n such that 2n + 1 is dibisible by 7.
2. Let a, b, c be the sides of a triangle. Prove that
a2 (b + c − a) + b2 (c + a − b) + c2 (a + b − c) ≤ 3abc

3. A circle is inscribed in triangle 4ABC with sides a, b, c. Tangents to the circle


parallel to the sides of the triangle are constructed. Each of these tangents cuts off a
triangle from 4ABC. In each of these triangle, a circle is inscribed. Find the sum
of the areas of all four inscribed circles (in terms of a, b, c)
4. Seventeen people correspond by mail with one another, each one with all the rest.
In their letters only three different topics are discussed. Each pair of correspondent
deals with only one of these topics. Prove that there are at least three people who
write to each other about the same topic.
5. Suppose five points in a plane are situated so that no two of the straight lines joining
the other four points. Determine the maximum number of intersections that these
perpendiculars can have.
6. In tetrahedron ABCD, vertex D is connected with D0 the centroid of 4ABC. Lines
parallel to DD0 are drawn through A, B and C. These lines intersect the planes
BCD, CAD and ABD in points A1 , B1 and C1 , respectively. Prove that the volume
of ABCD is one third the volume of A1 B1 C1 D0 . Is the result true if point D0 is
selected anywhere within 4ABC?

1.7 7th IMO, West Germany, 1965


1. Determine all value x in the interval 0 ≤ x ≤ 2π which satisfy the inequality



2 cos x ≤ 1 + sin 2x − 1 − sin 2x ≤ 2
12 CHAPTER 1. INTERNATIONAL MATHEMATICS OLYMPIAD

2. Consider the system of equations

a11 x1 + a12 x2 + a13 x3 = 0


a21 x1 + a22 x2 + a23 x3 = 0
a31 x1 + a32 x2 + a33 x3 = 0

with unknowns x1 , x2 , x3 . The coefficient satisfy the conditions:

(a) a11 , a22 , a33 are positive numbers;


(b) the remaining coefficients are negative numbers;
(c) in each equation, the sum of the coefficient is positive .

Prove that the given system has only the solution x1 = x2 = x3 = 0.

3. Given the tetrahedron ABCD whose edges AB and CD have lenght a and b respec-
tively. The distance between the skew lines AB and CD is d, and the angle between
them is ω. Tetrahedron ABCD is divided into two solid by plane ε, parallel to lines
AB and CD. The ratio of the distances of ε from AB and CD is equal to k. Compute
the ratio of the volumes of the two solids obtained.

4. Find all sets of four real numbers x1 , x2 , x3 , x4 such that the sum of any one and
the product of the other three is equal to 2.

5. Consider 4OAB with acute angle ^AOB. Through a point M 6= O perpendiculars


are dawn to OA and OB, the feet of which are P and Q respectively. The point of
intersection of the altitudes of 4OP Q is H. What is the locus of H if M is permitted
to range over

(a) the side AB?


(b) the interior of 4OAB?

6. In a plane a set of n points (n ≥ 3) is given. Each pair of points is connected by a


segment. Let d be the length of the longest of these segment. We define a diameter of
the set to be any connecting segment of length d. Prove that the number of diameters
of the given set is at most n.

1.8 8th IMO, Bulgaria, 1966


1. In a mathematical contest, three problems, A, B, C were posed. Among the par-
ticipants there were 25 students who solved at least one problem each. Of all the
1.9. 9T H IMO, YUGOSLAVIA, 1967 13

contestants who did not solve problem A, the number who solved B was twice the
number who solved C. The number of students who solved only problem A was one
more than the number of students who solved A and at least one other problem. How
many students solved only problem B?

2. Let a, b, c be the lengths of the sides of a triangle and α, β, γ, respectively, the


angles opposite these sides. Prove tat if a + b = tan γ2 (a tan α + b tan β), the triangle
is isosceles.

3. Prove: The sum of the distances of the vertices of a regular tetrahedron from the
centre of its circumscribed sphere is less than the sum of the distances of these vertices
from any other poin in space.

4. Prove that for every natural number n, and for every real number x 6= 2t
(t any
non-negative integer and k any integer),

1 1 1
+ +···+ = cot x − cot 2n x
sin 2x sin 4x sin 2n x

5. Solve the system of equations

|a1 − a2 |x2 + |a1 − a3 |x3 + |a1 − a4 |x4 = 1


|a2 − a1 |x2 + |a2 − a3 |x3 + |a2 − a4 |x4 = 1
|a3 − a1 |x1 + |a3 − a2 |x2 + |a3 − a4 |x4 = 1
|a4 − a1 |x1 + |a4 − a2 |x2 + |a4 − a3 |x3 = 1

where a1 , a2 , a3 , a4 are four different real numbers.

6. In the interior of sides BC, CA, AB of triangle 4ABC, any points K, L, M , respec-
tively, are selected. Prove that the area of at least one of the triangle 4AM L, 4BKM , 4CLK
is less than or equal to one quarter of the area of 4ABC

1.9 9th IMO, Yugoslavia, 1967


1. Let ABCD be a parallelogram with side lengths AB = a, AD = 1, and with ^BAD =
α. If 4ABD is acute, prove that the four circles of√radius 1 with centers A, B, C, D
cover the parallelogram if and only if a ≤ cos α + 3 sin α.

2. Prove that if one and only one edge of a tetrahedron is greater than 1, then its volume
is smaller than or equal to 81
14 CHAPTER 1. INTERNATIONAL MATHEMATICS OLYMPIAD

3. Let k, m, n be natural numbers such that m + k + 1 is a prime greater than n + 1.


Let cs = s (s + 1). Prove that the product (cm+1 − ck ) (cm+2 − ck ) · · · (cm+n − ck ) is
divisible by the product c1 c2 · · · cn .

4. Let 4A0 B0 C0 and 4A1 B1 C1 be any two acute-angled triangles. Consider all triangles
4ABC that are similar to 4A1 B1 C1 and circumscribed about triangle 4A0 B0 C0
(where A0 lies on BC, B0 on CA and C0 on AB) Of all such triangles, determine the
one with maximum area, and construct it.

5. Consider the sequence {cn }, where

c1 = a 1 + a 2 + · · · + a 8
c2 = a21 + a22 + · · · + a28
..
.
cn = an1 + an2 + · · · + an8
..
.

in which a1 , a2 , . . . , a8 are real numbers not all equal to zero. Suppose that an infinite
number of terms of the sequence {cn } are equal to zero. Find all natural numbers for
which cn = 0.

6. In a sport contest, there were m medals awarded on n successive days (n > 1). On
the first day, one medal and 71 of the remaining medals were awarded. On the second
day, two medals and 71 of the now remaining medals were awarded; and so on. On
the n-th and last day, the remaining n medals were awarded. How many days did
the contest last. and how many medals were awarded altogether?

1.10 10th IMO, USSR, 1968

1. Prove that there is one and only one triangle whose side lengths are consecutive
integers, and one of whose angles is twice as large as another.

2. Find all natural numbers x such that the product of their digits (in decimal notation)
is equal to x2 − 10x − 22.
1.10. 10T H IMO, USSR, 1968 15

3. Consider the system of equations:

ax21 + bx1 + c = x2
ax22 + bx2 + c = x3
..
.
ax2n−1 + bxn−1 + c = xn
ax2n + bxn + c = x1

with unknowns x1 , x2 , . . . , xn , where a, b, c are real and a 6= 0. Let 4 = (b − 1)2 −


4ac. Prove that for this system

(a) If 4 < 0, ther is no solution,


(b) If 4 = 0, ther is exactly one solution,
(c) If 4 > 0, ther is more than one solution.

4. Prove than in every tetrahedon there is a vertex such that the three edges meeting
there have lengths which are the sides of a triangle.

5. Let f be a real-valued function defined for all real numbers x such that, for some
positive constant a, the equation

1 q
f (x + a) = + f (x) − [f (x)]2
2

holds for all x

(a) Prove that the function f is periodic (i.e. there exists a positive number b such
that f (x + b) = f (x) for all x)
(b) For a = 1, give an example of a non-constant function with the requiered prop-
erties.

6. For every natural number n, evaluate the sum


$ % $ %
n + 2k n + 2k
   
X n+1 n+2
= + + · · · + +···
k=0 2k+1 2 4 2k+1

(the symbol bxc denotes the greatest integer not exceding x).
16 CHAPTER 1. INTERNATIONAL MATHEMATICS OLYMPIAD

1.11 11th IMO, Romania, 1969


1. Prove that there are infinitely many numbers a with the following property: the
number z = n4 + a is not prime for any natural number n.

2. Let a1 , a2 , . . . , an be real variable, and


1 1 1
f (x) = cos (a1 + x) + cos (a2 + x) + cos (a3 + x) + · · · + n−1 cos (an + x)
2 4 2
Given that f (x1 ) = f (x2 ) = 0, prove that x2 − x1 = mπ for some integer m.

3. For each value of k = 1, 2, 3, 4, 5, find necessary and sufficient conditions on the


number a > 0 so that there exist a tetrahedron with k edges of length a, and the
remaining 6 − k edges of lenght 1.

4. A semicircular arc γ is drawn on AB as diameter. C is a point on γ other than A and


B, and D is the foot of the perpendicular from C to AB. We consider three circles
γ1 , γ2 , γ3 , all tangent to the line AB. Of these, γ1 is inscrived in 4ABC, while γ2
and γ3 are both tangent to CD and to γ, one on each side of CD. Prove that γ1 , γ2
and γ3 have a second tangent in common.

5. Given n > 4 points


 in the plane such that no three are collinear. Prove that there
n−3
are at least 2 convex quadrilaterals whose vertices are four of the given points.

6. Prove that for all real numbers x1 , x2 , y1 , y2 , z1 , z2 with x1 > 0, x2 > 0, x1 y1 − z12 >
0, x2 y2 − z22 > 0, the inequality
8 1 1
2 ≤ +
(x1 + x2 ) (y1 + y2 ) − (z1 + z2 ) x1 y1 − z1 x2 y2 − z22
2

is satisfied. Give necessary and sufficient conditions for equality.

1.12 12th IMO, Hungary, 1970


1. Let M be a point on the sede AB of 4ABC. Let r1 , r2 and r be the radii of the
inscribed circles of the triangles 4AM C, 4BM C and 4ABC. Let q1 , q2 and q be
the radii of the excribed circles of the same triangles that lie in the angle 4ACB.
Prove that
r1 r2 r
· =
q1 q2 q
1.12. 12T H IMO, HUNGARY, 1970 17

2. Let a, b and n be integers greater than 1, and let a and b be the two bases of two
number systems. An−1 and An are numbers in the system with base a and Bn−1 and
Bn are numbers in the system with base b; these are related as follows:

An = xn xn−1 · · · x0 An−1 = xn−1 xn−2 · · · x0


Bn = xn xn−1 · · · x0 Bn−1 = xn−1 xn−2 · · · x0

such that xn 6= 0 and xn−1 6= 01 . Prove that

An−1 Bn−1
< ⇐
⇒a > b
An Bn

3. The real numbers a0 , a1 , . . . , an , . . . satisfy the condition 1 = a0 ≤ a1 ≤ a2 ≤ · · · ≤


an ≤ ·. The numbers b1 , b2 , . . . , bn , . . . are defined by
n  
X ak−1 1
bn = 1− √
k=1 ak ak

(a) Prove that 0 ≤ bn < 2 for all n.


(a) Given c with 0 ≤ c < 2, prove that there exist numbers a0 , a1 , . . . such that
bn > c for large enough n.

4. Find the set of all positive integers n with the property that the set {n, n + 1, n +
2, n + 3, n + 4, n + 5} can be partitioned into sets such that the product of the
numbers in one set equals the product of the numbers in the other set

5. In the tetrahedron ABCD, the angle ^BDC is a right angle. Suppose that the foot
H of the perpendicular from D to the plane ABC is the intersection of the altitudes
of 4ABC. Prove that
 
(AB + BC + CA)2 ≤ 6 AD 2 + BD 2 + CD 2

For what tetrahedra does equality hold?

6. In the plane are 100 points, no three of them are collinear. Consider all posible
triangles having these points as vertices. Prove that no more than 70% of these
triangles are acute-angled.
1
The xi ’s are the digits in the respective bases, and of course, all of them are lower than the lowest base
18 CHAPTER 1. INTERNATIONAL MATHEMATICS OLYMPIAD

1.13 13th IMO, Czechoslovakia, 1971


1. Prove that the following assertion is true for n = 3 and n = 5, and that it is false
for every other natural number n > 2. If a1 , a2 , . . . , an are arbitrary real numbers,
then:
(a1 − a2 ) (a1 − a3 ) · · · (a1 − an ) + (a2 − a1 ) (a2 − a3 ) · · · (a2 − an )
+ · · · + (an − a1 ) (an − a2 ) · · · (an − an−1 ) ≤ 0

2. Consider a convex polyhedron P1 with nine vertices A1 A2 , ..., A9 ; let Pi be the poly-
hedron obtained from P1 by a translation that moves vertex A1 to Ai (i = 2, 3, ..., 9).
Provethat at least two of the polyhedra P1 , P2 , ..., P9 have an interiorpoint in common.

3. Prove that the set of integers of the form 2k − 3(k = 2, 3, ...) contains an infinite
subset in which every two members are relatively prime.

4. All the faces of tetrahedron ABCD are acute-angled triangles. We consider all closed
polygonal paths of the form XY ZT X defined as follows: X is a point on edge AB
distinct from A and B; similarly, Y, Z, T are interior points of edges BCCD, DA,
respectively. Prove:
(a) If ^DAB + ^BCD 6= ^CDA + ^ABC, then among thepolygonal paths, there is
none of minimal length.
(b) If ^DAB + ^BCD = ^CDA + ^ABC, then there areinfinitely many shortest
polygonal paths, their common length being 2AC sin(α/2), where α = ^BAC +
^CAD + ^DAB.

5. Prove that for every natural number m, there exists a finite set S of points in a plane
with the following property: For every point A in S, there are exactly m points in S
which are at unit distance from A.

6. Let A = (aij )(i, j = 1, 2, ..., n) be a square matrix whose elements are non-negative
integers. Suppose that whenever an element aij = 0, the sum of the elements in the
ith row and the jth column is ≥ n. Prove that the sum of all the elements of the
matrix is ≥ n2 /2.

1.14 14th IMO, USSR, 1972


1. Prove that from a set of ten distinct two-digit numbers (in the decimalsystem), it is
possible to select two disjoint subsets whose members havethe same sum.
1.15. 15T H IMO, USSR, 1973 19

2. Prove that if n ≥ 4, every quadrilateral that can be inscribed in acircle can be


dissected into n quadrilaterals each of which is inscribablein a circle.

3. Let m and n be arbitrary non-negative integers. Prove that


(2m)!(2n)!
m!n!(m + n)!

is an integer. (0! = 1)

4. Find all solutions (x1 , x2 , x3 , x4 , x5 ) of the system of inequalities

(x21 − x3 x5 )(x22 − x3 x5 ) ≤ 0
(x22 − x4 x1 )(x23 − x4 x1 ) ≤ 0
(x23 − x5 x2 )(x24 − x5 x2 ) ≤ 0
(x24 − x1 x3 )(x25 − x1 x3 ) ≤ 0
(x25 − x2 x4 )(x21 − x2 x4 ) ≤ 0

where x1 , x2 , x3 , x4 , x5 are positive real numbers.

5. Let f and g be real-valued functions defined for all real values of xand y, and satisfying
the equation
f (x + y) + f (x − y) = 2f (x)g(y)

for all x, y. Prove that if f (x) is not identically zero, and if |f (x)| ≤ 1 for all x, then
|g(y)| ≤ 1 for all y.

6. Given four distinct parallel planes, prove that there exists a regular tetrahedron with
a vertex on each plane.

1.15 15th IMO, USSR, 1973


−−→ −−→ −−→
1. Point O lies on line g; OP1 , OP2 , . . . , OPn are unit vectors such that points P1 , P2 , ..., Pn
all lie in a plane containing g and on one side of g. Prove that if n is odd,
−−→ −−→ −−→
OP1 + OP2 + · · · + OPn ≥1

−−→ −−→

Here OM denotes the length of vector OM .
20 CHAPTER 1. INTERNATIONAL MATHEMATICS OLYMPIAD

2. Determine whether or not there exists a finite set M of points in spacenot lying in
the same plane such that, for any two points A and B of M,one can select two other
points C and D of M so that lines AB and CD are parallel and not coincident.

3. Let a and b be real numbers for which the equation

x4 + ax3 + bx2 + ax + 1 = 0

has at least one real solution. For all such pairs (a, b), find the minimum value of
a2 + b 2 .

4. A soldier needs to check on the presence of mines in a region having theshape of an


equilateral triangle. The radius of action of his detector isequal to half the altitude of
the triangle. The soldier leaves from one vertex of the triangle. What path should he
follow in order to travel the least possible distance and still accomplish his mission?

5. G is a set of non-constant functions of the real variable x of the form f (x) = ax + b, a


and b are real numbers, and G has the following properties:
(a) If f and g are in G, then g ◦ f is in G; here (g ◦ f )(x) = g[f (x)].
(b) If f is in G, then its inverse f −1 is in G; here the inverse of f (x) = ax + b is
f −1 (x) = (x − b)/a.
(c) For every f in G, there exists a real number xf such that f (xf ) = xf .
Prove that there exists a real number k such that f (k) = k for all f in G.

6. Let a1 , a2 , ..., an be n positive numbers, and let q be a givenreal number such that
0 < q < 1. Find n numbers b1 , b2 , ..., bn forwhich
(a) ak < bk for k = 1, 2, · · · , n,
bk+1 1
(b) q < bk
< q
for k = 1, 2, ..., n − 1,
1+q
(c) b1 + b2 + · · · + bn < 1−q
(a1 + a2 + · · · + an ).

1.16 16th IMO, West Germany, 1974


1. Three players A, B and C play the following game: On each of three cardsan integer
is written. These three numbers p, q, r satisfy 0 < p < q < r. Thethree cards are
shuffled and one is dealt to each player. Each then receivesthe number of counters
indicated by the card he holds. Then the cards areshuffled again; the counters remain
with the players.
1.17. 17T H IMO, BULGARIA, 1975 21

This process (shuffling, dealing, giving out counters) takes place for at least two
rounds. After the last round, A has 20 counters in all, B has 10 and C has 9. At the
last round B received r counters. Who received q counters on the first round?

2. In the triangle 4ABC prove that there is a point D on side AB suchthat CD is the
geometric mean of AD and DB if and only if
C
sin A sin B ≤ sin2 .
2
 
Pn 2n+1
3. Prove that the number k=0 2k+1
23k is not divisible by 5 for any integer n ≥ 0.

4. Consider decompositions of an 8 × 8 chessboard into p non-overlapping rectangles


subject to the following conditions:
(i) Each rectangle has as many white squares as black squares.
(ii) If ai is the number of white squares in the i-th rectangle, then a1 < a2 < · · · < ap .
Find the maximum value of p for which such a decomposition is possible. For this
value of p, determine all possible sequences a1 , a2 , · · · , ap .

5. Determine all possible values of


a b c d
S= + + +
a+b+d a+b+c b+c+d a+c+d

where a, b, c, d are arbitrary positive numbers.

6. Let P be a non-constant polynomial with integer coefficients. If n(P ) isthe number


of distinct integers k such that (P (k))2 = 1, prove that n(P ) − deg(P ) ≤ 2, where
deg(P ) denotes the degree of the polynomial P.

1.17 17th IMO, Bulgaria, 1975


1. Let xi , yi (i = 1, 2, ..., n) be real numbers such that

x1 ≥ x2 ≥ · · · ≥ xn and y1 ≥ y2 ≥ · · · ≥ yn

Prove that, if z1 , z2 , · · · , zn is any permutation of y1 , y2 , · · · , yn , then


n
X n
X
(xi − yi )2 ≤ (xi − zi )2
i=1 i=1
22 CHAPTER 1. INTERNATIONAL MATHEMATICS OLYMPIAD

2. Let a1 , a2 , a3, · · · be an infinite increasing sequence of positive integers. Prove that for
every p ≥ 1 there are infinitely many am which can be written in the form

am = xap + yaq

with x, y positive integers and q > p.

3. On the sides of an arbitrary triangle ABC, triangles ABR, BCP, CAQ areconstructed
externally with ^CBP = ^CAQ = 45◦ , ^BCP = ^ACQ = 30◦ , ^ABR = ^BAR =
15◦ . Prove that ^QRP = 90◦ and QR = RP.

4. When 44444444 is written in decimal notation, the sum of its digits is A. Let B be
the sum of the digits of A. Find the sum of the digits of B. (A and B are written in
decimal notation.)

5. Determine, with proof, whether or not one can find 1975 points on the circumference
of a circle with unit radius such that the distance between any two of them is a
rational number.

6. Find all polynomials P, in two variables, with the following properties:


(i) for a positive integer n and all real t, x, y

P (tx, ty) = tn P (x, y)

(that is, P is homogeneous of degree n),


(ii) for all real a, b, c,

P (b + c, a) + P (c + a, b) + P (a + b, c) = 0

(iii) P (1, 0) = 1.

1.18 18th IMO, Austria, 1976


1. In a plane convex quadrilateral of area 32, the sum of the lengths of two opposite
sides and one diagonal is 16. Determine all possible lengths ofthe other diagonal.

2. Let P1 (x) = x2 − 2 and Pj (x) = P1 (Pj−1 (x)) for j = 2, 3, · · ·.Show that, for any
positive integer n, the roots of the equation Pn (x) = x are real and distinct.
1.19. 19T H IMO, YUGOSLAVIA, 1977 23

3. A rectangular box can be filled completely with unit cubes. If one places as many
cubes as possible, each with volume 2, in the box, so that their edges are parallel
to the edges of the box, one can fill exactly 40% ofthe box. Determine the possible
dimensions of all such boxes.

4. Determine, with proof, the largest number which is the product of positiveintegers
whose sum is 1976.

5. Consider the system of p equations in q = 2p unknowns x1 , x2 , · · · , xq :

a11 x1 + a12 x2 + · · · + a1q xq = 0


a21 x1 + a22 x2 + · · · + a2q xq = 0
···
ap1 x1 + ap2 x2 + · · · + apq xq = 0

with every coefficient aij member of the set {−1, 0, 1}. Prove that the system has a
solution (x1 , x2 , · · · , xq ) such that
(a) all xj (j = 1, 2, ..., q) are integers,
(b) there is at least one value of j for which xj 6= 0,
(c) |xj | ≤ q(j = 1, 2, ..., q).

6. A sequence {un } is defined by

u0 = 2, u1 = 5/2, un+1 = un (u2n−1 − 2) − u1 for n = 1, 2, · · ·

Prove that for positive integers n,


n −(−1)n ]/3
[un ] = 2[2

where [x] denotes the greatest integer ≤ x.

1.19 19th IMO, Yugoslavia, 1977


1. Equilateral triangles 4ABK, 4BCL, 4CDM, 4DAN are constructed inside the
square ABCD. Prove that the midpoints of the four segments KL, LM, M N, N K
and the midpoints of the eight segments AK, BK, BL, CL, CM, DM, DN, AN
are the twelve vertices of a regular dodecagon.
24 CHAPTER 1. INTERNATIONAL MATHEMATICS OLYMPIAD

2. In a finite sequence of real numbers the sum of any seven successive terms is nega-
tive, and the sum of any eleven successive terms is positive.Determine the maximum
number of terms in the sequence.

3. Let n be a given integer > 2, and let Vn be the set of integers 1+kn, where k = 1, 2, ....
A number m ∈ Vn is called indecomposable in Vn if there do not exist numbers
p, q ∈ Vn such that pq = m. Prove that there exists a number r ∈ Vn that can be
expressed as the product of elements indecomposable in Vn in more than one way.
(Products which differ only in the order of their factors will be considered the same.)

4. Four real constants a, b, A, B are given, and

f (θ) = 1 − a cos θ − b sin θ − A cos 2θ − B sin 2θ

Prove that if f (θ) ≥ 0 for all real θ, then

a2 + b2 ≤ 2 and A2 + B 2 ≤ 1.

5. Let a and b be positive integers. When a2 + b2 is divided by a + b,the quotient is q


and the remainder is r. Find all pairs (a, b) suchthat q 2 + r = 1977.

6. Let f (n) be a function defined on the set of all positive integers and having all its
values in the same set. Prove that if f (n + 1) > f (f (n)) for each positive integer n,
then f (n) = n for each n.

1.20 20th IMO, Romania, 1978


1. m and n are natural numbers with 1 ≤ m < n. In their decimal representations, the
last three digits of 1978m are equal, respectively, to the last three digits of 1978n .
Find m and n such that m + n has its least value.

2. P is a given point inside a given sphere. Three mutually perpendicular rays from P
intersect the sphere at points U, V, and W ; Q denotes the vertex diagonally opposite
to P in the parallelepiped determined by P U, P V, and P W. Find the locus of Q for
all such triads of rays from P .

3. The set of all positive integers is the union of two disjoint subsets {f (1), f (2), ..., f (n), ...},
{g(1), g(2), ..., g(n), ...}, where

f (1) < f (2) < · · · < f (n) < · · ·


1.21. 21ST IMO, UNITED KINGDOM, 1979 25

g(1) < g(2) < · · · < g(n) < · · ·


and
g(n) = f (f (n)) + 1 for all n ≥ 1
Determine f (240).

4. In triangle 4ABC, AB = AC. A circle is tangent internally to thecircumcircle of


triangle ABC and also to sides AB, AC at P, Q, respectively. Prove that the
midpoint of segment P Q is the center of the incircle of triangle 4ABC.

5. Let {ak }(k = 1, 2, 3, ..., n, ...) be a sequence of distinct positive integers. Prove that
for all natural numbers n,
n n
X ak X 1
2

k=1 k k=1 k

6. An international society has its members from six different countries. The list of
members contains 1978 names, numbered 1, 2, ..., 1978. Prove that there is at least
one member whose number is the sum of thenumbers of two members from his own
country, or twice as large as the numberof one member from his own country.

1.21 21st IMO, United Kingdom, 1979


1. Let p and q be natural numbers such that
p 1 1 1 1 1
= 1− + − +···− +
q 2 3 4 1318 1319
Prove that p is divisible by 1979.

2. A prism with pentagons A1 A2 A3 A4 A5 and B1 B2 B3 B4 B5 as top and bottom faces is


given. Each side of the two pentagons and each of the line-segments Ai Bj for all
i, j = 1, ..., 5, is colored either red or green. Every triangle whose vertices are vertices
of the prism and whose sides have all been colored has two sides of a different color.
Show that all 10 sides of the top and bottom faces are the same color.

3. Two circles in a plane intersect. Let A be one of the points of intersection. Starting
simultaneously from A two points move with constant speeds, each point travelling
along its own circle in the same sense. The two points return to A simultaneously
after one revolution. Prove that there is a fixed point P in the plane such that, at
any time, the distances from P to the moving points are equal.
26 CHAPTER 1. INTERNATIONAL MATHEMATICS OLYMPIAD

4. Given a plane π, a point P in this plane and a point Q not in π, find all points R in
π such that the ratio (QP + P A)/QR is a maximum.

5. Find all real numbers a for which there exist non-negative real numbers x1 , x2 , x3 , x4 , x5
satisfying the relations
5
X 5
X 5
X
kxk = a, k 3 xk = a 2 , k 5 xk = a 3
k=1 k=1 k=1

6. Let A and E be opposite vertices of a regular octagon. A frog starts jumping at


vertex A. From any vertex of the octagon except E, it may jump to either of the two
adjacent vertices. When it reaches vertex E, the frog stops and stays there.. Let a n
be the number of distinct paths of exactly n jumps ending at E. Prove that a2n−1 = 0,
1
a2n = √ (xn−1 − y n−1 ), n = 1, 2, 3, · · · ,
2
√ √
where x = 2 + 2 and y = 2 − 2
Note. A path of n jumps is a sequence of vertices (P0 , ..., Pn ) such that
(i) P0 = A, Pn = E;
(ii) for every i, 0 ≤ i ≤ n − 1, Pi is distinct from E;
(iii) for every i, 0 ≤ i ≤ n − 1, Pi and Pi+1 are adjacent.

1.22 22nd IMO, Washington, USA, 1981


1. P is a point inside a given triangle ABC.D, E, F are the feet of the perpendiculars
from P to the lines BC, CA, AB respectively. Find all P for which
BC CA AB
+ +
PD PE PF
is least.

2. Let 1 ≤ r ≤ n and consider all subsets of r elements of theset {1, 2, ..., n}. Each
of these subsets has a smallest member. Let F (n, r) denote the arithmetic mean of
these smallest numbers; prove that
n+1
F (n, r) =
r+1
1.23. 23RD IMO, BUDAPEST, HUNGARY, 1982 27

3. Determine the maximum value of m3 + n3 ,where m and n are integers satisfying


m, n ∈ {1, 2, ..., 1981} and (n2 − mn − m2 )2 = 1

4. (a) For which values of n > 2 is there a set of n consecutive positive integers such
that the largest number in the set is a divisor of the least common multiple of the
remaining n − 1 numbers?
(b) For which values of n > 2 is there exactly one set having the stated property?

5. Three congruent circles have a common point O and lie inside a given triangle. Each
circle touches a pair of sides of the triangle. Prove that the incenter and the circum-
center of the triangle and the point O are collinear.

6. The function f (x, y) satisfies


(1) f (0, y) = y + 1,
(2)f (x + 1, 0) = f (x, 1),
(3) f (x + 1, y + 1) = f (x, f (x + 1, y)),
for all non-negative integers x, y. Determine f (4, 1981).

1.23 23rd IMO, Budapest, Hungary, 1982


1. The function f (n) is defined for all positive integers n and takes on non-negative
integer values. Also, for all m, n

f (m + n) − f (m) − f (n) = 0 or 1

f (2) = 0, f (3) > 0, and f (9999) = 3333


Determine f (1982).

2. A non-isosceles triangle A1 A2 A3 is given with sides a1 , a2 , a3 (ai is the side opposite


Ai ). For all i = 1, 2, 3, Mi is the midpoint of side ai , and Ti . is the pointwhere the
incircle touches side ai . Denote by Si the reflection of Ti in the interior bisector of
angle Ai . Prove that the lines M1 , S1 , M2 S2 , and M3 S3 are concurrent.

3. Consider the infinite sequences {xn } of positive real numbers with the following prop-
erties:
x0 = 1, and for all i ≥ 0, xi+1 ≤ xi
28 CHAPTER 1. INTERNATIONAL MATHEMATICS OLYMPIAD

(a) Prove that for every such sequence, there is an n ≥ 1 such that
x20 x21 x2
+ + · · · + n−1 ≥ 3.999
x1 x2 xn
(b) Find such a sequence for which
x20 x21 x2
+ + · · · + n−1 < 4
x1 x2 xn

4. Prove that if n is a positive integer such that the equation x3 − 3xy 2 + y 3 = n has
a solution in integers (x, y), then it has at least three suchsolutions. Show that the
equation has no solutions in integers when n = 2891
5. The diagonals AC and CE of the regular hexagon ABCDEF are divided by the
inner points M and N , respectively, so that
AM CN
= =r
AC CE
Determine r if B, M, and N are collinear.
6. Let S be a square with sides of length 100, and let L be a path within S which does
not meet itself and which is composed of line segments A0 A1 , A1 A2 , · · · , An−1 An with
A0 6= An . Suppose that for every point P of the boundary of S there is a point of L
at a distance from P not greater than 1/2. Prove that there are two points X and Y
in L such that the distance between X and Y is not greater than 1, and the length
of that part of L which lies between X and Y is not smaller than 198.

1.24 24th IMO, Paris, France, 1983


1. Find all functions f defined on the set of positive real numbers which take positive
real values and satisfy the conditions:
(i) f (xf (y)) = yf (x) for all positive x, y;
(ii) f (x) → 0 as x → ∞
2. Let A be one of the two distinct points of intersection of two unequal coplanar circles
C1 and C2 with centers O1 and O2 , respectively. One of the common tangents to the
circles touches C1 at P1 and C2 at P2 , while the other touches C1 at Q1 and C2 at
Q2 . Let M1 be the midpoint of P1 Q1 ,and M2 be the midpoint of P2 Q2 . Prove that
^O1 AO2 = ^M1 AM2 .
1.25. 25T H IMO, PRAGUE, CZECHOSLOVAKIA, 1984 29

3. Let a, b and c be positive integers, no two of which have a common divisor greater
than 1. Show that 2abc − ab − bc − ca is the largest integer which cannot be expressed
in the form xbc + yca + zab,where x, y and z are non-negative integers.

4. Let ABC be an equilateral triangle and E the set of all points contained in the three
segments AB, BC and CA (including A, B and C). Determine whether, for every
partition of E into two disjoint subsets, at least one of the two subsets contains the
vertices of a right-angled triangle. Justify your answer.

5. Is it possible to choose 1983 distinct positive integers, all less than or equal to 10 5 ,
no three of which are consecutive terms of an arithmetic progression? Justify your
answer.

6. Let a, b and c be the lengths of the sides of a triangle. Prove that

a2 b(a − b) + b2 c(b − c) + c2 a(c − a) ≥ 0

Determine when equality occurs.

1.25 25th IMO, Prague, Czechoslovakia, 1984


1. Prove that
7
0 ≤ yz + zx + xy − 2xyz ≤
27
where x, y and z arenon-negative real numbers for which x + y + z = 1.

2. Find one pair of positive integers a and b such that:


(i) ab(a + b) is not divisible by 7;
(ii) (a + b)7 − a7 − b7 is divisible by 77 .
Justify your answer.

3. In the plane two different points O and A are given. For each point X of the plane,
other than O, denote by a(X) the measure of the angle between OA and OX in
radians, counterclockwise from OA(0 ≤ a(X) < 2π). Let C(X) be the circle with
center O and radius of length OX + a(X)/OX. Each point of the plane is colored by
one of a finite number ofcolors. Prove that there exists a point Y for which a(Y ) > 0
such that its color appears on the circumference of the circle C(Y ).
30 CHAPTER 1. INTERNATIONAL MATHEMATICS OLYMPIAD

4. Let ABCD be a convex quadrilateral such that the line CD is a tangent to the circle
on AB as diameter. Prove that the line AB is a tangent to the circle on CD as
diameter if and only if the lines BC and AD are parallel.

5. Let d be the sum of the lengths of all the diagonals of a plane convex polygon with
n vertices (n > 3), and let p be its perimeter. Prove that
  
2d n n+1
n−3< < −2
p 2 2
where [x] denotes the greatest integer not exceeding x.

6. Let a, b, c and d be odd integers such that 0 < a < b < c < d and ad = bc. Prove that
if a + d = 2k and b + c = 2m for some integers k and m, then a = 1.

1.26 26th IMO, Helsinki, Finland, 1985


1. A circle has center on the side AB of the cyclic quadrilateral ABCD. The other three
sides are tangent to the circle. Prove that AD + BC = AB

2. Let n and k be given relatively prime natural numbers, k < n. Each number in the
set M = {1, 2, ..., n − 1} is colored either blue or white. It is given that
(i) for each i ∈ M, both i and n − i have the same color;
(ii) for each i ∈ M, i 6= k both i and |i − k| have the same color. Prove that all
numbers in M must have the same color.

3. For any polynomial P (x) = a0 + a1 x + · · · + ak xk with integer coefficients, the number


of coefficients which are odd is denoted by w(P ). For i = 0, 1, ..., let Qi (x) = (1 + x)i .
Prove that if i1 i2 , ..., in are integers such that 0 ≤ i1 < i2 < · · · < in then

w(Qi1 + Qi2 + · · · + Qin ) ≥ w(Qi1 )

4. Given a set M of 1985 distinct positive integers, none of which has a prime divisor
greater than 26. Prove that M contains at least one subset of four distinct elements
whose product is the fourth power of an integer.

5. A circle with center O passes through the vertices A and C of triangle ABC and
intersects the segments AB and BC again at distinct points K and N, respectively.
The circumscribed circles of the triangles 4ABC and 4EBN intersect at exactly
two distinct points B and M. Provethat angle ^OM B is a right angle.
1.27. 27T H IMO, WARSAW, POLAND, 1986 31

6. For every real number x1 , construct the sequence x1 , x2 , ... by setting


 
1
xn+1 = xn xn + for each n ≥ 1
n
Prove that there exists exactly one value of x1 for which

0 < xn < xn+1 < 1

for every n.

1.27 27th IMO, Warsaw, Poland, 1986


1. Let d be any positive integer not equal to 2, 5, or 13. Show that one canind distinct
a, b in the set {2, 5, 13, d} such that ab − 1 is not perfect square.

2. A triangle 4A1 A2 A3 and a point P0 are given in the plane. We define As = As−3 for
all s ≥ 4. We construct a set of points P1 , P2 , P3 , . . . , such that Pk+1 is the image
of Pk under a rotation withenter Ak+1 through angle 120◦ clockwise (for k = 0, 1,
2,ldots). Prove that if P1986 = P0 , then the triangle 4A1 A2 A3 equilateral.

3. To each vertex of a regular pentagon an integer is assigned in such a way that the sum
of all five numbers is positive. If three consecutive vertices are assigned the numbers
x, y, z respectively and y < 0 then the following operation is allowed: the numbers
x, y, z are replaced by x + y, −y, z + y respectively. Such an operation is performed
repeatedly as long as at least one of the five numbers is negative. Determine whether
this procedure necessarily comes to and end after a finite number of steps.

4. Let A, B be adjacent vertices of a regular n-gon (n ≥ 5) in thelane having center at


O. A triangle XY Z, which is congruent to andnitially conincides with OAB, moves
in the plane in such a way that Y and Z each trace out the whole boundary of the
polygon, X remaining inside the polygon. Find the locus of X.

5. Find all functions f , defined on the non-negative real numbers and taking non-
negative real values, such that:

(i) f (xf (y))f (y) = f (x + y) for all x, y ≥ 0,


(ii) f (2) = 0,
(iii) f (x) 6= 0 for 0 ≤ x < 2.
32 CHAPTER 1. INTERNATIONAL MATHEMATICS OLYMPIAD

6. One is given a finite set of points in the plane, each point having integeroordinates.
Is it always possible to color some of the points in the set rednd the remaining points
white in such a way that for any straight line Larallel to either one of the coordinate
axes the difference (in absolutealue) between the numbers of white point and red
points on L is not greaterhan 1?

1.28 28th IMO, Havana, Cuba , 1987


1. Let pn (k) be the number of permutations of the set {1, . . . , n}, n ≥ 1, which have
exactly k fixed points. Prove that
n
X
k · pn (k) = n!
k=0

(Remark: A permutation f of a set S is a one-to-one mapping of S onto itself. An


element i in S is called a fixed point of the permutation f if f (i) = i.)

2. In an acute-angled triangle 4ABC the interior bisector of the angle ^A intersects


BC at L and intersects the circumcircle of 4ABC again at N . From point L per-
pendiculars are drawn to AB and AC, the feet of theseerpendiculars being K and M
respectively. Prove that the quadrilateral AKN M and the triangle ABC have equal
areas.

3. Let x1 , x2 , . . . , xn be real numbers satisfying x21 + x22 + · · · + x2n = 1. Prove that for
every integer k ≥ 2 there are integers a1 , a2 , . . . , an , not all 0, such that |ai | ≤ k − 1
For all i and √
(k − 1) n
|a1 x1 + a1 x2 + · · · + an xn | ≤
kn −

4. Prove that there is no function f from the set of non-negative integers into itself such
that f (f (n)) = n + 1987 for every n.

5. Let n be an integer greater than or equal to 3. Prove that there is a set of n points
in the plane such that the distance between any two points is irrational and each set
of three points determines a non-degenerate triangle with rational area.

or equal to 2. Prove that if k 2 + k + n is prime for


6. Let n be an integer greater thanq
all integers k such that 0 ≤ k ≤ n3 , then k 2 + k + n is prime for all integers k such
that 0 ≤ k ≤ n − 2.
1.29. 29T H IMO, CAMBERRA, AUSTRALIA, 1988 33

1.29 29th IMO, Camberra, Australia, 1988


1. Consider two coplanar circles of radii R and r (R > r) with the same center. Let P
be a fixed point on the smaller circle and B a variable point on the larger circle. The
line BP meets the larger circle again at C. The perpendicular l to BP at P meets
the smaller circle again at A. (If l is tangent to the circle at P then A = P .)
(i) Find the set of values of BC 2 + CA2 + AB 2 .
(ii) Find the locus of the midpoint of BC.
2. Let n be a positive integer and let A1 , A2 , . . . , A2n+1 be subsets of a set B. Suppose
that
(a) Each Ai has exactly 2n elements,
(b) Each Ai ∩ Aj (1 ≤ i < j ≤ 2n + 1) contains exactly one element, and
(c) Every element of B belongs to at least two of the Ai .
For which values of n can one assign to every element of B one of the numbers 0 and
1 in such a way that Ai has 0 assigned to exactly n of its elements?
3. A function f is defined on the positive integers by
f (1) = 1, f (3) = 3,
f (2n) = f (n),
f (4n + 1) = 2f (2n + 1) − f (n),
f (4n + 3) = 3f (2n + 1) − 2f (n),
for all positive integers n.
Determine the number of positive integers n, less than or equal to 1988, for which
f (n) = n.
4. Show that set of real numbers x which satisfy the inequality
70
X k 5

k=1 x − k 4
is a union of disjoint intervals, the sum of whose lengths is 1988.
5. 4ABC is a triangle right-angled at A, and D is the foot of the altituderom A. The
straight line joining the incenters of the triangles 4ABD, 4ACD intersects the sides
AB, AC at the points K, L respectively. S and T denote the areas of the triangles
4ABC and 4AKL respectively.how that S ≥ 2T .
34 CHAPTER 1. INTERNATIONAL MATHEMATICS OLYMPIAD

6. Let a and b be positive integers such that ab + 1 divides a2 + b2 . Show that


a2 + b 2
ab + 1
is the square of an integer.

1.30 30th IMO, Braunschweig, West Germany, 1989


1. Prove that the set {1, 2, . . . , 1989} can be expressed as the disjoint union of subsets
Ai (i = 1, 2, . . . , 117) such that:
(i) Each Ai contains 17 elements;
(ii) The sum of all the elements in each Ai is the same.
2. In an acute-angled triangle ABC the internal bisector of angle A meets their cumcircle
of the triangle again at A1 . Points B1 and C1 are defined similarly. Let A0 be the
point of intersection of the line AA1 with the external bisectors of angles B and C.
Points B0 and C0 are defined similarly. Prove that:
(i) The area of the triangle A0 B0 C0 is twice the area of the hexagon AC1 BA1 CB1 .
(ii) The area of the triangle A0 B0 C0 is at least four times the area of the triangle
ABC.
3. Let n and k be positive integers and let S be a set of n points in the plane such that
(i) No three points of S are collinear, and
(ii) For any point P of S there are at least k points of S equidistant from P .
Prove that:
1 √
k< + 2n
2
4. Let ABCD be a convex quadrilateral such that the sides AB, AD, BC satisfy AB =
AD + BC. There exists a point P inside the quadrilateral at a distance h from the
line CD such that AP = h + AD and BP = h + BC. Show that:
1 1 1
√ ≥√ +√
h AD BC

5. Prove that for each positive integer n there exist n consecutive positive integers none
of which is an integral power of a prime number.
1.30. 30T H IMO, BRAUNSCHWEIG, WEST GERMANY, 1989 35

6. A permutation (x1 , x2 , . . . , xm ) of the set {1, 2, . . . , 2n}, where n is a positive integer,


is said to have property P if |xi − xi+1 | = n for at least one i in {1, 2, . . . , 2n − 1}.
Show that, for each n, there are more permutations with property P than without.
36 CHAPTER 1. INTERNATIONAL MATHEMATICS OLYMPIAD

1.31 31st IMO, Beijing, People’s Republic of China,


1990
1. Chords AB and CD of a circle intersect at a point E inside the circle. Let M be an
interior point of the segment EB. The tangent line at E to the circle through D, E,
and M intersects the lines BC and AC at F and G, respectively. If
AM
= t,
AB
find
EG
EF
in terms of t.
2. Let n ≥ 3 and consider a set E of 2n − 1 distinct points on a circle. Suppose that
exactly k of these points are to be colored black. Such a coloring is “good” if there
is at least one pair of black points such that the interior of one of the arcs between
them contains exactly n points from E. Find the smallest value of k so that every
such coloring of k points of E is good.
3. Determine all integers n > 1 such that
2n + 1
n2
is an integer.
4. Let Q+ be the set of positive rational numbers. Construct a function f : Q+ → Q+
such that
f (x)
f (xf (y)) =
y
+
for all x, y in Q .
5. Given an initial integer n0 > 1, two players, A and B, choose integers n1 , n2 , n3 ,
. . . alternately according to the following rules:
Knowing n2k , A chooses any integer n2k+1 such that

n2k ≤ n2k+1 ≤ n22k .

Knowing n2k+1 , B chooses any integer n2k+2 such that


n2k+1
n2k+2
1.32. 32N D IMO, SIGTUNA, SWEDEN, 1991 37

is a prime raised to a positive integer power.


Player A wins the game by choosing the number 1990; player B wins by choosing the
number 1. For which n0 does:
(a) A have a winning strategy?
(b) B have a winning strategy?
(c) Neither player have a winning strategy?
6. Prove that there exists a convex 1990-gon with the following two properties:
(a) All angles are equal.
(b) The lengths of the 1990 sides are the numbers 12 , 22 , 32 , . . . , 19902 in some
order.

1.32 32nd IMO, Sigtuna, Sweden, 1991


1. Given a triangle ABC, let I be the center of its inscribed circle. The internal
bisectors of the angles A, B, C meet the opposite sides in A0 , B 0 , C 0 respectively.
Prove that
1 AI · BI · CI 8
< 0 0 0

4 AA · BB · CC 27
2. Let n > 6 be an integer and a1 , a2 , . . . , ak be all the natural numbers less than n
and relatively prime to n. If

a2 − a1 = a3 − a2 = · · · = ak − ak−1 > 0

prove that n must be either a prime number or a power of 2.


3. Let S = {1, 2, 3, . . . , 280}. Find the smallest integer n such that each n-element
subset of S contains five numbers which are pairwise relatively prime.
4. Suppose G is a connected graph with k edges. Prove that it is possible to label
the edges 1, 2, . . . , k in such a way that at each vertex which belongs to two or more
edges, the greatest common divisor of the integers labeling those edges is equal to 1.
[A graph consists of a set of points, called vertices, together with a set of edges
joining certain pairs of distinct vertices. Each pair of vertices u, v belongs to at
most one edge. The graph G is connected if for each pair of distinct vertices x, y
there is some sequence of vertices x = v0 , v1 , v2 , . . . , vm = y such that each pair
vi , vi+1 (0 ≤ i < m) is joined by an edge of G.]
38 CHAPTER 1. INTERNATIONAL MATHEMATICS OLYMPIAD

5. Let 4ABC be a triangle and P an interior point of 4ABC. Show that at least one
of the angles ^P AB, ^P BC, ^P CA is less than or equal to 30◦ .
6. An infinite sequence x0 , x1 , x2 , . . . of real numbers is said to be bounded if there is a
constant C such that |xi | ≤ C for every i ≥ 0.
Given any real number a > 1, construct a bounded infinite sequence x0 , x1 , x2 , . . .
such that
|xi − xj ||i − j|a ≥ 1
for every pair of distinct nonnegative integers i, j.

1.33 33rd IMO, Moscow, Russia, 1992


1. Find all integers a, b, c with 1 < a < b < c such that
(a − 1)(b − 1)(c − 1) is a divisor of abc − 1

2. Let R denote the set of all real numbers. Find all functions f : R → R such that
 
f x2 + f (y) = y + (f (x))2 forall x, y ∈ R

3. Consider nine points in space, no four of which are coplanar. Each pair of points is
joined by an edge (that is, a line segment) and each edge is either colored blue or red
or left uncolored. Find the smallest value of n such that whenever exactly n edges
are colored, the set of colored edges necessarily contains a triangle all of whose edges
have the same color.
4. In the plane let C be a circle, L a line tangent to the circle C, and M a point
on L. Find the locus of all points P with the following property: there exists two
points Q, R on L such that M is the midpoint of QR and C is the inscribed circle
of triangle 4P QR.
5. Let S be a finite set of points in three-dimensional space. Let Sx , Sy , Sz be the
sets consisting of the orthogonal projections of the points of S onto the yz-plane,
zx-plane, xy -plane, respectively. Prove that

|S|2 ≤ |Sx | · |Sy | · |Sz |


where |A| denotes the number of elements in the finite set |A|. (Note: The orthogonal
projection of a point onto a plane is the foot of the perpendicular from that point to
the plane.)
1.34. 34T H IMO, ISTAMBUL, TURKEY, 1993 39

6. For each positive integer n, S(n) is defined to be the greatest integer such that, for
every positive integer k ≤ S(n), n2 can be written as the sum of k positive squares.

(a) Prove that S(n) ≤ n2 − 14 for each n ≥ 4.


(b) Find an integer n such that S(n) = n2 − 14.
(c) Prove that there are infintely many integers n such that S(n) = n2 − 14.

1.34 34th IMO, Istambul, Turkey, 1993


1. Let f (x) = xn + 5xn−1 + 3, where n > 1 is an integer. Prove that f (x) cannot be
expressed as the product of two nonconstant polynomials with integer coefficients.

2. Let D be a point inside acute triangle ABC such that ∠ADB = ^ACB + π/2 and
AC · BD = AD · BC.

(a) Calculate the ratio (AB · CD)/(AC · BD).


(b) Prove that the tangents at C to the circumcircles of 4ACD and 4BCD are
perpendicular.

3. On an infinite chessboard, a game is played as follows. At the start, n2 pieces are


arranged on the chessboard in an n by n block of adjoining squares, one piece in each
square. A move in the game is a jump in a horizontal or vertical direction over an
adjacent occupied square to an unoccupied square immediately beyond. The piece
which has been jumped over is removed.
Find those values of n for which the game can end with only one piece remaining on
the board.

4. For three points P, Q, R in the plane, we define m(P QR) as the minimum length of
the three altitudes of 4P QR. (If the points are collinear, we set m(P QR) = 0.)
Prove that for points A, B, C, X in the plane,

m(ABC) ≤ m(ABX) + m(AXC) + m(XBC)

5. Does there exist a function f : N → N such that f (1) = 2, f (f (n)) = f (n) + n for
all n ∈ N, and f (n) < f (n + 1) for all n ∈ N?
40 CHAPTER 1. INTERNATIONAL MATHEMATICS OLYMPIAD

6. There are n lamps L0 , . . . , Ln−1 in a circle (n > 1), where we denote Ln+k = Lk . (A
lamp at all times is either on or off.) Perform steps s0 , s1 , . . . as follows: at step si , if
Li−1 is lit, switch Li from on to off or vice versa, otherwise do nothing. Initially all
lamps are on. Show that:

(a) There is a positive integer M (n) such that after M (n) steps all the lamps are
on again;
(b) If n = 2k , we can take M (n) = n2 − 1;
(c) If n = 2k + 1, we can take M (n) = n2 − n + 1.

1.35 35th IMO, Hong Kong, 1994


1. Let m and n be positive integers. Let a1 , a2 , . . . , am be distinct elements of {1, 2, . . . , n}
such that whenever ai +aj ≤ n for some i, j, 1 ≤ i ≤ j ≤ m, there exists k, 1 ≤ k ≤ m,
with ai + aj = ak . Prove that

a1 + a 2 + · · · + a m n+1

m 2

2. 4ABC is an isosceles triangle with AB = AC. Suppose that

(a) M is the midpoint of BC and O is the point on the line AM such that OB is
perpendicular to AB;
(b) Q is an arbitrary point on the segment BC different from B and C;
(c) E lies on the line AB and F lies on the line AC such that E , Q, F are distinct
and collinear.

Prove that OQ is perpendicular to EF if and only if QE = QF .

3. For any positive integer k, let f (k) be the number of elements in the set {k + 1, k +
2, . . . , 2k} whose base 2 representation has precisely three 1s.

(a) Prove that, for each positive integer m, there exists at least onepositive integer
k such that f (k) = m.
(b) Determine all positive integers m for which there exists exactly one k with
f (k) = m.
1.36. 36T H IMO, TORONTO, CANADA, 1995 41

4. Determine all ordered pairs (m, n) of positive integers such that

n3 + 1
mn − 1
is an integer.

5. Let S be the set of real numbers strictly greater than −1. Find all functions f : S → S
satisfying the two conditions:

(a) f (x + f (y) + xf (y)) = y + f (x) + yf (x) for all x and y in S;


f (x)
(b) x
is strictly increasing on each of the intervals −1 < x < 0 and 0 < x.

6. Show that there exists a set A of positive integers with the following property: For
any infinite set S of primes there exist two positive integers m ∈ A and n ∈
/ A each
of which is a product of k distinct elements of S for some k ≥ 2.

1.36 36th IMO, Toronto, Canada, 1995


1. Let A, B, C, D be four distinct points on a line, in that order. The circles with
diameters AC and BD intersect at X and Y . The line XY meets BC at Z. Let
P be a point on the line XY other than Z. The line CP intersects the circle with
diameter AC at C and M , and the line BP intersects the circle with diameter BD
at B and N . Prove that the lines AM, DN, XY are concurrent.

2. Let a, b, c be positive real numbers such that abc = 1. Prove that


1 1 1 3
+ 3 + 3 ≥
a3 (b+ c) b (c + a) c (a + b) 2

3. Determine all integers n > 3 for which there exist n points A1 , . . . , An in the plane,
no three collinear, and real numbers r1 , . . . , rn such that for 1 ≤ i < j < k ≤ n, the
area of 4Ai Aj Ak is ri + rj + rk .

4. Find the maximum value of x0 for which there exists a sequence x0 , x1 . . . , x1995 of
positive reals with x0 = x1995 , such that for i = 1, . . . , 1995,
2 1
xi−1 + = 2xi +
xi−1 xi
42 CHAPTER 1. INTERNATIONAL MATHEMATICS OLYMPIAD

5. Let ABCDEF be a convex hexagon with AB = BC = CD and DE = EF = F A,


such that ^BCD = ^EF A = π/3. Suppose G and H are points in theinterior of the
hexagon such that ^AGB = ^DHE = 2π/3. Provethat AG + GB + GH + DH +
HE ≥ CF .

6. Let p be an odd prime number. How many p-element subsets A of {1, 2, . . . 2p} are
there, the sum of whose elements is divisible by p?

1.37 37th IMO, Mumbai, India, 1996


1. We are given a positive integer r and a rectangular board ABCD with dimensions
|AB| = 20, |BC| = 12. The rectangle is divided into a grid of 20 × 12 unit squares.
The following moves are permitted on the board: one can move from√one square to
another only if the distance between the centers of the two squares is r. The task is
to find a sequence of moves leading from the square with A as a vertex to the square
with B as a vertex.

(a) Show that the task cannot be done if r is divisible by 2 or 3.


(b) Prove that the task is possible when r = 73.
(c) Can the task be done when r = 97?

2. Let P be a point inside triangle 4ABC such that

^AP B − ^ACB = ^AP C − ^ABC

Let D, E be the incenters of triangles AP B, AP C, respectively. Show that AP, BD, CE


meet at a point.

3. Let S denote the set of nonnegative integers. Find all functions f from S to itself
such that
f (m + f (n)) = f (f (m)) + f (n) ∀m, n ∈ S

4. The positive integers a and b are such that the numbers 15a + 16b and 16a − 15b are
both squares of positive integers. What is the least possible value that can be taken
on by the smaller of these two squares?

5. Let ABCDEF be a convex hexagon such that AB is parallel to DE, BC is parallel


to EF , and CD is parallel to F A. Let RA , RC , RE denote the circumradii of trian-
gles F AB, BCD, DEF , respectively, and let P denote the perimeter of the hexagon.
1.38. 38T H IMO, MAR DEL PLATA, ARGENTINA, 1997 43

Prove that
P
RA + R C + R E ≥
2
6. Let p, q, n be three positive integers with p + q < n. Let (x0 , x1 , . . . , xn ) be an (n + 1)-
tuple of integers satisfying the following conditions:

(a) x0 = xn = 0.
(b) For each i with 1 ≤ i ≤ n, either xi − xi−1 = p or xi − xi−1 = −q.

Show that there exist indices i < j with (i, j) 6= (0, n), such that xi = xj .

1.38 38th IMO, Mar del Plata, Argentina, 1997


1. In the plane the points with integer coordinates are the vertices of unit squares. The
squares are colored alternately black and white (as on a chessboard). For any pair of
positive integers m and n, consider a right-angled triangle whose vertices have integer
coordinates and whose legs, of lengths m and n,ie along edges of the squares.
Let S1 be the total area of the black part of the triangle and S2 be the total area of
thehite part. Let
f (m, n) = |S1 − S2 |

(a) Calculate f (m, n) for all positive integers m and n which are eitheroth even or
both odd.
1
(b) Prove that f (m, n) ≤ 2
max{m, n} for all m and n.
(c) Show that there is no constant C such that f (m, n) < C for all m and n.
endenumerate

2. The angle at A is the smallest angle of triangle 4ABC. The points B and C divide
the circumcircle of the triangle into two arcs. Let U be an interior point of the arc
between B and C which does not contain A. The perpendicular bisectors of AB and
AC meet the line AU at V and W , respectively. The lines BV and CW meet at T .
Show that
AU = T B + T C

3. Let x1 , x2 , . . . , xn be real numbers satisfying the conditions

|x1 + x2 + · · · + xn | = 1
44 CHAPTER 1. INTERNATIONAL MATHEMATICS OLYMPIAD

and
n+1
|xi | ≤ i = 1, 2, . . . , n
2
Show that there exists a permutation y1 , y2 , . . . , yn of x1 , x2 , . . . , xn such that
n+1
|y1 + 2y2 + · · · + nyn | ≤
2

4. An n × n matrix whose entries come from the set S = {1, 2, . . . , 2n − 1} is called


a silver matrix if, for each i = 1, 2, . . . , n, the ith row and the ith column together
contain all elements of S. Show that
(a) there is no silver matrix for n = 1997;
(b) silver matrices exist for infinitely many values of n.
5. Find all pairs (a, b) of integers a, b ≥ 1 that satisfy the equation
2
ab = b a

6. For each positive integer n , let f (n) denote the number of ways of representing n as
a sum of powers of 2 with nonnegative integer exponents.epresentations which differ
only in the ordering of their summands are considered to be the same. For instance,
f (4) = 4, because the number 4 can be represented in the following four ways:

4; 2 + 2; 2 + 1 + 1; 1 + 1 + 1 + 1.

Prove that, for any integer n ≥ 3,


2 /4 2 /2
2n < f (2n ) < 2n .

1.39 39th IMO, Taipei, Taiwan, 1998


1. In the convex quadrilateral ABCD, the diagonals AC and BD are perpendicular and
the opposite sides AB and DC are not parallel. Suppose that the point P , where the
perpendicular bisectors of AB and DC meet, is inside ABCD. Prove that ABCD is
a cyclic quadrilateral if and only if the triangles ABP and CDP have equal areas.
2. In a competition, there are a contestants and b judges, where b ≥ 3 is an odd integer.
Each judge rates each contestant as either pass or fail. Suppose k is a number such
that, for any two judges, their ratings coincide for at most k contestants. Prove that
k/a ≥ (b − 1)/(2b).
1.40. 40T H IMO, BUCHAREST, ROMANIA, 1999 45

3. For any positive integer n, let d(n) denote the number of positive divisors of n (in-
cluding 1 and n itself). Determine all positive integers k such that d(n2 )/d(n) = k
for some n.

4. Determine all pairs (a, b) of positive integers such that ab2 + b + 7 divides a2 b + a + b.

5. Let I be the incenter of triangle ABC. Let the incircle of ABC touch the sides BC,
CA, and AB at K, L, and M , respectively. The line through B parallel to M K
meets the lines LM and LK at R and S, respectively. Prove that angle RIS is acute.

6. Consider all functions f from the set N of all positive integers into itself satisfying
f (t2 f (s)) = s(f (t))2 for all s and t in N . Determine the least possible value of
f (1998).

1.40 40th IMO, Bucharest, Romania, 1999


1. Determine all finite sets S of at least three points in the plane which satisfy the
following condition:

for any two distinct points A and B in S, the perpendicular bisector of the
line segment AB is an axis of symmetry for S.

2. Let n be a fixed integer, with n ≥ 2.

(a) Determine the least constant C such that the inequality


 4
X X
xi xj (x2i + x2j ) ≤ C  xi 
1≤i<j≤n 1≤i≤n

holds for all real numbers x1 , · · · , xn ≥ 0.

(b) For this constant C, determine when equality holds.

3. Consider an n × n square board, where n is a fixed even positive integer. The board
is divided into n2 unit squares. We say that two different squares on the board are
adjacent if they have a common side.
N unit squares on the board are marked in such a way that every square (marked or
unmarked) on the board is adjacent to at least one marked square.
Determine the smallest possible value of N .
46 CHAPTER 1. INTERNATIONAL MATHEMATICS OLYMPIAD

4. Determine all pairs (n, p) of positive integers such that


p is a prime,
n not exceeded 2p, and
(p − 1)n + 1 is divisible by np−1 .
5. Two circles G1 and G2 are contained inside the circle G, and are tangent to G at the
distinct points M and N , respectively. G1 passes through the center of G2 . The line
passing through the two points of intersection of G1 and G2 meets G at A and B.
The lines M A and M B meet G1 at C and D, respectively.
Prove that CD is tangent to G2 .
6. Determine all functions f : R −→ R such that
f (x − f (y)) = f (f (y)) + xf (y) + f (x) − 1
for all real numbers x, y.

1.41 41st IMO, Taejon, South Korea, 2000


1. Two circles ω1 and ω2 intersect at M and N . Line ` is tangent to the circles at A
and B, respectively, so that M lies closer to ` than N . Line CD, with C on ω1 and
D on ω2 , is parallel to ` and passes through M . Let lines AC and BD meet at E;
let lines AN and CD meet at P ; and let lines BN and CD meet at Q. Prove that
EP = EQ.
2. Let a, b, c be positive real numbers such that abc = 1. Prove that:
   
1 1 1
a−1+ b−1+ c−1+ ≤1
b c a

3. Let n ≥ 2 be a positive integer. Initially, there are n fleas on a horizontal line, not
all at the same point.

For a positive real number λ, define a move as follows:


choose any two fleas at points A and B, with A to the left of B; let the flea
at A jump to the point C on the line to the right of B with BCAB
= λ.

Determine all the values of λ such that for any point M on the line and any initial
position of the n fleas, there is a sequence of moves that will take all the fleas to the
position to the right of M .
1.42. 42N D IMO, WASHINGTON DC, USA, 2001 47

4. A magician has one hundred cards numbered 1 to 100. He puts them into three boxes,
a red one, a white one and a blue one, so that each box contain at least one card.
A member of the audience selects two of the three boxes, choose one card from each
and announces the sum of the numbers on the chosen cards. Given this sum, the
magician identifies the box from which no card has been choosen.
How many ways are there to put all the cards into the boxes so that this trick always
works? (Two ways are considered diferent if at least one of card is put nto a different
box)
5. Determine whether or not there exists a positive integer n such that:
n is divisible by exactely 2000 different prime numbers, and
2n + 1 is divisible by n.
6. Let AH1 , BH2 , CH3 be the altitudes of an acute-angled triangle 4ABC. The incircle
of the triangle 4ABC touches the sides BC, CA, AB at T1 , T2 , T3 , respectively. Let
the lines `1 , `2 , `3 be the reflections of the lines H2 H3 , H3 H1 H1 H2 in the lines T2 T3 ,
T3 T1 , T1 T2 , respectively.
Prove that `1 , `2 , `3 determine a triangle whose vertices lie on the incircle of the
triangle 4ABC.

1.42 42nd IMO, Washington DC, USA, 2001


1. Let 4ABC be an acute-angled triangle with circumcentre O. Let P on BC be the
foot of the altitude from A. Suppose that ^BCA ≥ ^ABC + 30◦ . Prove that
^CAB + ^COP < 90◦
2. Prove that
a b c
√ +√ 2 +√ 2 ≥1
a2
+ 8bc b + 8ca c + 8ab
for all positive real numbers a, b and c.
3. Twenty-one girls and twenty-one boys took part in a mathematical contest.
• Each contestant solved at most six problems.
• For each girl and each boy, at least one problem was solved by both of them
Prove that there was a problem that was solved by at least three girls and at least
three boys.
48 CHAPTER 1. INTERNATIONAL MATHEMATICS OLYMPIAD

4. Let n be an odd integer greater than 1, and let k1 , k2 , . . . , kn be given integers. For
each of the n! permutations a = (a1 , a2 , . . . , an ) of 1, 2, . . . , n, let
n
X
S (a) = ki ai
i=1

Prove that there are two permutations b and c, such that n! is a divisor of S (b)−S (c).

5. In a triangle 4ABC, let AP bisect ^BAC, with P on BC, and let BQ bisect ^ABC,
with Q on CA. It is known that ^BAC = 60◦ and that AB + BP = AQ + QB.
What are the possible angles of triangle 4ABC?

6. Let a, b, c, d be integers with a > b > c > d > 0. Suppose that

ac + bd = (b + d + a − c) (b + d − a + c)

Prove ab + cd is not a prime.

1.43 43rd IMO, Glascow, United Kingdom, 2002


1. S is the set of all (h, k) with h, k non-negative integers such that h + k < n. Each
element of S is colored red or blue, so that if (h, k) is red and h0 ≤ h, k 0 ≤ k, then
(h0 , k 0 ) is also red. A type 1 subset of S has n blue elements with different first
member and a type 2 subset of S has n blue elements with different second member.
Show that there are the same number of type 1 and type 2 subsets.

2. BC is a diameter of a circle center O. A is any point on the circle with ∠AOC >
60o . EF is the chord which is the perpendicular bisector of AO. D is the midpoint of
the minor arc AB. The line through O parallel to AD meets AC at J. Show that J
is the incenter of triangle CEF .

3. Find all pairs of integers m > 2, n > 2 such that there are infinitely many positive
integers k for which k n + k 2 − 1 divides k m + k − 1.

4. The positive divisors of the integer n > 1 are d1 < d2 < . . . < dk , so that d1 = 1, dk =
n. Let d = d1 d2 + d2 d3 + · · · + dk−1 dk .

(a) Prove that D < n2 .


(b) Determine all n for which D is a divisor of n2 .
1.44. 44T H IMO, TOKYO, JAPAN, 2003 49

5. Find all functions f from the set R of real numbers to itself such that
(f (x) + f (z))(f (y) + f (t)) = f (xy − zt) + f (xt + yz)
for all x, y, z, t in R.
6. n > 2 circles of radius 1 are drawn in the plane so that no line meets more than two
of the circles. Their centers are O1 , O2 , · · · , On . Show that 1≤i<j≤n Oi1Oj ≤ (n−1)π
P
4

1.44 44th IMO, Tokyo, Japan, 2003


1. Let A be a 101-element subset of the set S = {1, 2, . . . , 1000000}. Prove that there
exist numbers t1 , t2 , . . . , t100 in S such that the sets
Aj = {x + tj | x ∈ A} j = 1, 2, . . . , 100
are pairwise disjoint.
m2
2. Find all pairs (m, n) of positive integers such that 2mn2 −n3 +1
is a positive integer.
3. A convex hexagon is given in which any two
√ opposite sides have the following property:
the distance between their midpoints is 3/2 times the sum of their lengths. Prove
that all the angles of the hexagon are equal.
(A convex ABCDEF has three pairs of opposite sides: AB and DE, BC and EF ,
CD and F A.)
4. Let ABCD be a cyclic quadrilateral. Let P, Q and R be the feet of perpendiculars
from D to lines BC, CA and AB, respectively. Showhat P Q = QR if and only if the
bisectors of angles ^ABC and ^ADC meet on segment AC.
5. Let n be a positive integer and x1 , x2 , . . . , xn be real numbers with x1 ≤ x2 ≤ . . . ≤ xn .
(a) Prove that
 2
n X
n n X n
X 2(n2 − 1) X
 |xi − x j | ≤ (xi − xj )2
i=1 j=1 3 i=1 j=1

(b) Show that the equality holds if and only if x1 , x2 , . . . , xn form an arithmetic
sequence.
6. Show that for each prime p, there exists a prime q such that np − p is not divisible
by q for any positive integer n.
Chapter 2

William Lowell Putnam Competition

2.1 46th Anual William Lowell Putnam Competition,


1985
1. Determine, with proof, the number of ordered triples (A1 , A2 , A3 ) of sets which have
the property that
(i) A1 ∪ A2 ∪ A3 = {1, 2, 3, 4, 5, 6, 7, 8, 9, 10}, and
(ii) A1 ∩ A2 ∩ A3 = ∅.
Express your answer in the form 2a 3b 5c 7d , where a, b, c, d are nonnegative integers.
2. Let T be an acute triangle. Inscribe a rectangle R in T with one side along a side of
T . Then inscribe a rectangle S in the triangle formed by the side of R opposite the
side on the boundary of T , and the other two sides of T , with one side along the side
of R. For any polygon X, let A(X) denote the area of X. Find the maximum value,
or show that no maximum exists, of A(R)+A(S)
A(T )
, where T ranges over all triangles and
R, S over all rectangles as above.
3. Let d be a real number. For each integer m ≥ 0, define a sequence {am (j)}, j =
0, 1, 2, . . . by the condition
am (0) = d/2m ,
am (j + 1) = (am (j))2 + 2am (j), j ≥ 0.

Evaluate limn→∞ an (n).

50
2.1. 46T H ANUAL WILLIAM LOWELL PUTNAM COMPETITION, 1985 51

4. Define a sequence {ai } by a1 = 3 and ai+1 = 3ai for i ≥ 1. Which integers between
00 and 99 inclusive occur as the last two digits in the decimal expansion of infinitely
many ai ?
R 2π
5. Let Im = 0 cos(x) cos(2x) · · · cos(mx) dx. For which integers m, 1 ≤ m ≤ 10 is
Im 6= 0?

6. If p(x) = a0 + a1 x + · · · + am xm is a polynomial with real coefficients ai , then set

Γ(p(x)) = a20 + a21 + · · · + a2m .

Let F (x) = 3x2 + 7x + 2. Find, with proof, a polynomial g(x) with real coefficients
such that

(i) g(0) = 1, and


(ii) Γ(f (x)n ) = Γ(g(x)n )

for every integer n ≥ 1.

7. Let k be the smallest positive integer for which there exist distinct integers m1 , m2 , m3 , m4 , m5
such that the polynomial

p(x) = (x − m1 )(x − m2 )(x − m3 )(x − m4 )(x − m5 )

has exactly k nonzero coefficients. Find, with proof, a set of integers m1 , m2 , m3 , m4 , m5


for which this minimum k is achieved.

8. Define polynomials fn (x) for n ≥ 0 by f0 (x) = 1, fn (0) = 0 for n ≥ 1, and


d
fn+1 (x) = (n + 1)fn (x + 1)
dx
for n ≥ 0. Find, with proof, the explicit factorization of f100 (1) into powers of distinct
primes.

9. Let
a1,1 a1,2 a1,3 ...
a2,1 a2,2 a2,3 ...
a3,1 a3,2 a3,3 ...
.. .. .. ..
. . . .
be a doubly infinite array of positive integers, and suppose each positive integer
appears exactly eight times in the array. Prove that am,n > mn for some pair of
positive integers (m, n).
52 CHAPTER 2. WILLIAM LOWELL PUTNAM COMPETITION

10. Let C be the unit circle x2 +y 2 = 1. A point p is chosen randomly on the circumference
C and another point q is chosen randomly from the interior of C (these points are
chosen independently and uniformly over their domains). Let R be the rectangle with
sides parallel to the x and y-axes with diagonal pq. What is the probability that no
point of R lies outside of C?
R R ∞ −x2 √
11. Evaluate 0∞ t−1/2 e−1985(t+t ) dt. You may assume that −∞
−1
e dx = π.

12. Let G be a finite set of real n × n matrices {Mi }, 1 ≤ i ≤ r, which form a group
P
under matrix multiplication. Suppose that ri=1 tr(Mi ) = 0, where tr(A) denotes the
Pr
trace of the matrix A. Prove that i=1 Mi is the n × n zero matrix.

2.2 47th Anual William Lowell Putnam Competition,


1986
1. Find, with explanation, the maximum value of f (x) = x3 − 3x on the set of all real
numbers x satisfying x4 + 36 ≤ 13x2 .

2. What is the units (i.e., rightmost) digit of


$ %
1020000
?
10100 + 3

P
3. Evaluate ∞ 2
n=0 Arccot(n + n + 1), where Arccot t for t ≥ 0 denotes the number θ in
the interval 0 < θ ≤ π/2 with cot θ = t.

4. A transversal of an n × n matrix A consists of n entries of A, no two in the same row


or column. Let f (n) be the number of n × n matrices A satisfying the following two
conditions:

(a) Each entry αi,j of A is in the set {−1, 0, 1}.


(b) The sum of the n entries of a transversal is the same for all transversals of A.

An example of such a matrix A is


 
−1 0 −1
 
A =  0 1 0 .
0 1 0
2.2. 47T H ANUAL WILLIAM LOWELL PUTNAM COMPETITION, 1986 53

Determine with proof a formula for f (n) of the form


f (n) = a1 bn1 + a2 bn2 + a3 bn3 + a4 ,
where the ai ’s and bi ’s are rational numbers.
5. Suppose f1 (x), f2 (x), . . . , fn (x) are functions of n real variables x = (x1 , . . . , xn ) with
continuous second-order partial derivatives everywhere on Rn . Suppose further that
there are constants cij such that
∂fi ∂fj
− = cij
∂xj ∂xi
for all i and j, 1 ≤ i ≤ n, 1 ≤ j ≤ n. Prove that there is a function g(x) on Rn such
that fi + ∂g/∂xi is linear for all i, 1 ≤ i ≤ n. (A linear function is one of the form
a0 + a1 x1 + a2 x2 + · · · + an xn .)

6. Let a1 , a2 , . . . , an be real numbers, and let b1 , b2 , . . . , bn be distinct positive integers.


Suppose that there is a polynomial f (x) satisfying the identity
n
X
n
(1 − x) f (x) = 1 + a i xb i .
i=1

Find a simple expression (not involving any sums) for f (1) in terms of b1 , b2 , . . . , bn
and n (but independent of a1 , a2 , . . . , an ).
7. Inscribe a rectangle of base b and height h in a circle of radius one, and inscribe
an isosceles triangle in the region of the circle cut off by one base of the rectangle
(with that side as the base of the triangle). For what value of h do the rectangle and
triangle have the same area?
8. Prove that there are only a finite number of possibilities for the ordered triple T =
(x − y, y − z, z − x), where x, y, z are complex numbers satisfying the simultaneous
equations
x(x − 1) + 2yz = y(y − 1) + 2zx + z(z − 1) + 2xy,
and list all such triples T .
9. Let Γ consist of all polynomials in x with integer coefficienst. For f and g in Γ and
m a positive integer, let f ≡ g (mod m) mean that every coefficient of f − g is an
integral multiple of m. Let n and p be positive integers with p prime. Given that
f, g, h, r and s are in Γ with rf + sg ≡ 1 (mod p) and f g ≡ h (mod p), prove
that there exist F and G in Γ with F ≡ f (mod p), G ≡ g (mod p), and F G ≡ h
(mod pn ).
54 CHAPTER 2. WILLIAM LOWELL PUTNAM COMPETITION

10. For a positive real number r, let G(r) be the minimum value of |r − m2 + 2n2 | for
all integers m and n. Prove or disprove the assertion that limr→∞ G(r) exists and
equals 0.

11. Let f (x, y, z) = x2 + y 2 + z 2 + xyz. Let p(x, y, z), q(x, y, z), r(x, y, z) be polynomials
with real coefficients satisfying

f (p(x, y, z), q(x, y, z), r(x, y, z)) = f (x, y, z).

Prove or disprove the assertion that the sequence p, q, r consists of some permutation
of ±x, ±y, ±z, where the number of minus signs is 0 or 2.

12. Suppose A, B, C, D are n × n matrices with entries in a field F , satisfying the condi-
tions that AB T andCD T are symmetric and AD T − BC T = I. Here I is the n × n
identity matrix, and if M is an n × n matrix, M T is its transpose. Prove that
AT D + C T B = I.

2.3 48th Anual William Lowell Putnam Competition,


1987
1. Curves A, B, C and D are defined in the plane as follows:
( )
2 x2
A = (x, y) : x − y = 2 ,
x + y2
( )
y
B = (x, y) : 2xy + 2 =3 ,
x + y2
n o
C = (x, y) : x3 − 3xy 2 + 3y = 1 ,
n o
D = (x, y) : 3x2 y − 3x − y 3 = 0 .

Prove that A ∩ B = C ∩ D.

2. The sequence of digits

123456789101112131415161718192021 . . .

is obtained by writing the positive integers in order. If the 10n -th digit in this sequence
occurs in the part of the sequence in which the m-digit numbers are placed, define
f (n) to be m. For example, f (2) = 2 because the 100th digit enters the sequence in
the placement of the two-digit integer 55. Find, with proof, f (1987).
2.3. 48T H ANUAL WILLIAM LOWELL PUTNAM COMPETITION, 1987 55

3. For all real x, the real-valued function y = f (x) satisfies

y 00 − 2y 0 + y = 2ex .

(a) If f (x) > 0 for all real x, must f 0 (x) > 0 for all real x? Explain.
(b) If f 0 (x) > 0 for all real x, must f (x) > 0 for all real x? Explain.

4. Let P be a polynomial, with real coefficients, in three variables and F be a function


of two variables such that

P (ux, uy, uz) = u2 F (y − x, z − x) for all real x, y, z, u,

and such that P (1, 0, 0) = 4, P (0, 1, 0) = 5, and P (0, 0, 1) = 6. Also let A, B, C be


complex numbers with P (A, B, C) = 0 and |B − A| = 10. Find |C − A|.

5. Let !
~ −y x
G(x, y) = , ,0 .
x2 + 4y 2 x2 + 4y 2
Prove or disprove that there is a vector-valued function

F~ (x, y, z) = (M (x, y, z), N (x, y, z), P (x, y, z))

with the following properties:

(i) M, N, P have continuous partial derivatives for all (x, y, z) 6= (0, 0, 0);
(ii) Curl F~ = ~0 for all (x, y, z) 6= (0, 0, 0);
(iii) F~ (x, y, 0) = G(x,
~ y).

6. For each positive integer n, let a(n) be the number of zeroes in the base 3 represen-
tation of n. For which positive real numbers x does the series

X xa(n)
3
n=1 n

converge?

7. Evaluate q
Z 4 ln(9 − x) dx
q q .
2 ln(9 − x) + ln(x + 3)
56 CHAPTER 2. WILLIAM LOWELL PUTNAM COMPETITION

8. Let r, s and t be integers with 0 ≤ r, 0 ≤ s and r + s ≤ t. Prove that


    
s s s
0 1 s t+1

t
+  
t
+···+ 
t
 = 
t−s

r r+1 r+s
(t + 1 − s) r

9. Let F be a field in which 1 + 1 6= 0. Show that the set of solutions to the equation
x2 + y 2 = 1 with x and y in F is given by (x, y) = (1, 0) and
!
r 2 − 1 2r
(x, y) = ,
r2 + 1 r2 + 1

where r runs through the elements of F such that r 2 6= −1.

10. Let (x1 , y1 ) = (0.8, 0.6) and let xn+1 = xn cos yn − yn sin yn and yn+1 = xn sin yn +
yn cos yn for n = 1, 2, 3, . . .. For each of limn→∞ xn and limn→∞ yn , prove that the
limit exists and find it or prove that the limit does not exist.

11. Let On be the n-dimensional vector (0, 0, · · · , 0). Let M be a 2n × n matrix of


complex numbers such that whenever (z1 , z2 , . . . , z2n )M = On , with complex zi , not
all zero, then at least one of the zi is not real. Prove that for arbitrary real numbers
r1 , r2 , . . . , r2n , there are complex numbers w1 , w2 , . . . , wn such that
    
w1 r1
  .   .. 
re M  .. 
 
 = 
 
.
. 
wn rn

(Note: if C is a matrix of complex numbers, re(C) is the matrix whose entries are
the real parts of the entries of C.)

12. Let F be the field of p2 elements, where p is an odd prime. Suppose S is a set of
(p2 − 1)/2 distinct nonzero elements of F with the property that for each a 6= 0 in F ,
exactly one of a and −a is in S. Let N be the number of elements in the intersection
S ∩ {2a : a ∈ S}. Prove that N is even.

2.4 49th Anual William Lowell Putnam Competition,


1988
1. Let R be the region consisting of the points (x, y) of the cartesian plane satisfying
both |x| − |y| ≤ 1 and |y| ≤ 1. Sketch the region R and find its area.
2.4. 49T H ANUAL WILLIAM LOWELL PUTNAM COMPETITION, 1988 57

2. A not uncommon calculus mistake is to believe that the product rule for derivatives
2
says that (f g)0 = f 0 g 0 . If f (x) = ex , determine, with proof, whether there exists an
open interval (a, b) and a nonzero function g defined on (a, b) such that this wrong
product rule is true for x in (a, b).

3. Determine, with proof, the set of real numbers x for which


∞  x
X 1 1
csc −1
n=1 n n

converges.

4. (a) If every point of the plane is painted one of three colors, do there necessarily
exist two points of the same color exactly one inch apart?
(b) What if three is replaced by nine?

5. Prove that there exists a unique function f from the set R+ of positive real numbers
to R+ such that
f (f (x)) = 6x − f (x)
and
f (x) > 0
for all x > 0.

6. If a linear transformation A on an n-dimensional vector space has n + 1 eigenvectors


such that any n of them are linearly independent, does it follow that A is a scalar
multiple of the identity? Prove your answer.

7. A composite (positive integer) is a product ab with a and b not necessarily distinct


integers in {2, 3, 4, . . .}. Show that every composite is expressible as xy + xz + yz + 1,
with x, y, z positive integers.

8. Prove or disprove: If x and y are real numbers with y ≥ 0 and y(y + 1) ≤ (x + 1)2 ,
then y(y − 1) ≤ x2 .

9. For every√n in the set N = {1, 2, . . .} of positive integers, let rn be the minimum value
of |c − d 3| for all nonnegative integers c and d with c + d = n. Find, with proof,
the smallest positive real number g with rn ≤ g for all n ∈ N.
P
10. Prove that if ∞ n=1 an is a convergent series of positive real numbers, then so is
P∞ n/(n+1)
(a
n=1 n ) .
58 CHAPTER 2. WILLIAM LOWELL PUTNAM COMPETITION

11. For positive integers n, let Mn be the 2n + 1 by 2n + 1 skew-symmetric matrix for


which each entry in the first n subdiagonals below the main diagonal is 1 and each
of the remaining entries below the main diagonal is -1. Find, with proof, the rank
of Mn . (According to one definition, the rank of a matrix is the largest k such that
there is a k × k submatrix with nonzero determinant.)
One may note that  
0 −1 1

M1 =  1 0 −1 

−1 1 0
 
0 −1 −1 1 1
 


1 0 −1 −1 1 
M2 = 
 1 1 0 −1 −1 


 −1 1 1 0 −1 
−1 −1 1 1 0

12. Prove that there exist an infinite number of ordered pairs (a, b) of integers such that
for every positive integer t, the number at + b is a triangular number if and only if t
is a triangular number. (The triangular numbers are the tn = n(n + 1)/2 with n in
{0, 1, 2, . . .})

2.5 50th Anual William Lowell Putnam Competition,


1989
1. How many primes among the positive integers, written as usual in base 10, are alter-
nating 1’s and 0’s, beginning and ending with 1?
Z a Z b 2 x2 ,a2 y 2 }
2. Evaluate emax{b dy dx where a and b are positive.
0 0

3. Prove that if
11z 10 + 10iz 9 + 10iz − 11 = 0,
then |z| = 1. (Here z is a complex number and i2 = −1.)

4. If α is an irrational number, 0 < α < 1, is there a finite game with an honest coin
such that the probability of one player winning the game is α? (An honest coin is one
for which the probability of heads and the probability of tails are both 12 . A game is
finite if with probability 1 it must end in a finite number of moves.)
2.5. 50T H ANUAL WILLIAM LOWELL PUTNAM COMPETITION, 1989 59

5. Let m be a positive integer and let G be a regular (2m + 1)-gon inscribed in the unit
circle. Show that there is a positive constant A, independent of m, with the following
property. For any points p inside G there are two distinct vertices v1 and v2 of G such
that
1 A
| |p − v1 | − |p − v2 | | < − 3.
m m
Here |s − t| denotes the distance between the points s and t.

6. Let α = 1 + a1 x + a2 x2 + · · · be a formal power series with coefficients in the field of


two elements. Let


 1 if every block of zeros in the binary expansion of n

 has an even number of zeros in the block
an =




0 otherwise.

(For example, a36 = 1 because 36 = 1001002 and a20 = 0 because 20 = 101002 .)


Prove that α3 + xα + 1 = 0.
7. A dart, thrown at random, hits a square target. Assuming that any two parts of the
target of equal area are equally likely to be hit, find the probability that the point
√ hit
a b+c
is nearer to the center than to any edge. Express your answer in the form ,
d
where a, b, c, d are integers.

8. Let S be a non-empty set with an associative operation that is left and right can-
cellative (xy = xz implies y = z, and yx = zx implies y = z).ssume that for every a
in S the set {an : n = 1, 2, 3, . . .} is inite. Must S be a group?
9. Let f be a function on [0, ∞), differentiable and satisfying

f 0 (x) = −3f (x) + 6f (2x)



for x > 0. Assume that |f (x)| ≤ e− x for x ≥ 0 (so that f (x) tends rapidly to 0 as x
increases). For n a non-negative integer, define
Z ∞
µn = xn f (x) dx
0

(sometimes called the nth moment of f ).

a) Express µn in terms of µ0 .
n
b) Prove that the sequence {µn 3n! } always converges, and that the limit is 0 only
if µ0 = 0.
60 CHAPTER 2. WILLIAM LOWELL PUTNAM COMPETITION

10. Can a countably infinite set have an uncountable collection of non-empty subsets such
that the intersection of any two of them is finite?

11. Label the vertices of a trapezoid T (quadrilateral with two parallel sides) inscribed
in the unit circle as A, B, C, D so that AB is parallel to CD and A, B, C, D are
in counterclockwise order. Let s1 , s2 , and d denote the lengths of the line segments
AB, CD, and OE, where E is the point of intersection of the diagonals of T , and O
is the center of the circle. Determine the least upper bound of s1 −s d
2
over all such T
for which d 6= 0, and describe allases, if any, in which it is attained.

12. Let (x1 , x2 , . . . xn ) be a point chosen at random from the n-dimensional region defined
by 0 < x1 < x2 < · · · < xn < 1. Let f be a continuous function on [0, 1] with f (1) = 0.
Set x0 = 0 and xn+1 = 1. Show that the expected value of the Riemann sum
n
X
(xi+1 − xi )f (xi+1 )
i=0

R
is 01 f (t)P (t) dt, where P is a polynomial of degree n, independent of f , with 0 ≤
P (t) ≤ 1 for 0 ≤ t ≤ 1.

2.6 51th Anual William Lowell Putnam Competition,


1990
1. Let
T0 = 2, T1 = 3, T2 = 6,
and for n ≥ 3,
Tn = (n + 4)Tn−1 − 4nTn−2 + (4n − 8)Tn−3 .
The first few terms are

2, 3, 6, 14, 40, 152, 784, 5168, 40576.

Find, with proof, a formula for Tn of the form Tn = An + Bn , where {An } and {Bn }
are well-known sequences.
√ √ √
2. Is 2 the limit of a sequence of numbers of the form 3 n − 3 m (n, m = 0, 1, 2, . . .)?

3. Prove that any convex pentagon whose vertices (no three of which are collinear) have
integer coordinates must have area greater than or equal to 25 .
2.6. 51T H ANUAL WILLIAM LOWELL PUTNAM COMPETITION, 1990 61

4. Consider a paper punch that can be centered at any point of the plane and that,
when operated, removes from the plane precisely those points whose distance from
the center is irrational. How many punches are needed to remove every point?
5. If A and B are square matrices of the same size such that ABAB = 0, does it follow
that BABA = 0?
6. If X is a finite set, let X denote the number of elements in X. Call an ordered pair
(S, T ) of subsets of {1, 2, . . . , n} admissible if s > |T | for each s ∈ S, and t > |S| for
each t ∈ T . How many admissible ordered pairs of subsets of {1, 2, . . . , 10} are there?
Prove your answer.
7. Find all real-valued continuously differentiable functions f on the real line such that
for all x, Z x
2
(f (x)) = [(f (t))2 + (f 0 (t))2 ] dt + 1990.
0

8. Prove that for |x| < 1, |z| > 1,



X
1+ (1 + xj )Pj = 0,
j=1

where Pj is
(1 − z)(1 − zx)(1 − zx2 ) · · · (1 − zxj−1 )
.
(z − x)(z − x2 )(z − x3 ) · · · (z − xj )
9. Let S be a set of 2 × 2 integer matrices whose entries aij (1) are all squares of integers
and, (2) satisfy aij ≤ 200. Show that if S has more than 50387 (= 154 − 152 − 15 + 2)
elements, then it has two elements that commute.
10. Let G be a finite group of order n generated by a and b. Prove or disprove: there is
a sequence
g1 , g2 , g3 , . . . , g2n
such that
(1) every element of G occurs exactly twice, and
(2) gi+1 equals gi a or gi b for i = 1, 2, . . . , 2n. (Interpret g2n+1 as g1 .)
11. Is there an infinite sequence a0 , a1 , a2 , . . . of nonzero real numbers such that for n =
1, 2, 3, . . . the polynomial
pn (x) = a0 + a1 x + a2 x2 + · · · + an xn
has exactly n distinct real roots?
62 CHAPTER 2. WILLIAM LOWELL PUTNAM COMPETITION

12. Let S be a nonempty closed bounded convex set in the plane. Let K be a line and t
a positive number. Let L1 and L2 be support lines for S parallel to K1 , and let L be
the line parallel to K and midway between L1 and L2 . Let BS (K, t) be the band of
points whose distance from L is at most (t/2)w, where w is the distance between L1
and L2 . What is the smallest t such that
\
S∩ BS (K, t) 6= ∅
K

for all S? (K runs over all lines in the plane.)

2.7 52th Anual William Lowell Putnam Competition,


1991
1. A 2 × 3 rectangle has vertices as (0, 0), (2, 0), (0, 3), and (2, 3). It rotates 90 ◦ clockwise
about the point (2, 0). It then otates 90◦ clockwise about the point (5, 0), then 90◦
clockwise about the point (7, 0), and finally, 90◦ clockwise about the point (10, 0).
(The side originally on the x-axis is now back on the x-axis.) Find the area of the
region above the x-axis and below the curve traced out by the point whose initial
position is (1,1).

2. Let A and B be different n×n matrices with real entries. If A3 = B 3 and A2 B = B 2 A,


can A2 + B 2 be nvertible?

3. Find all real polynomials p(x) of degree n ≥ 2 for which there exist real numbers
r1 < r2 < · · · < rn such that

(a) p(ri ) = 0, i = 1, 2, . . . , n, and


 
ri +ri+1
(b) p0 2
=0 i = 1, 2, . . . , n − 1,

where p0 (x) denotes the derivative of p(x).

4. Does there exist an infinite sequence of closed discs D1 , D2 , D3 , . . . in the plane, with
centers c1 , c2 , c3 , . . ., respectively, such that

(a) the ci have no limit point in the finite plane,


(b) the sum of the areas of the Di is finite, and
(c) every line in the plane intersects at least one of the Di ?
2.7. 52T H ANUAL WILLIAM LOWELL PUTNAM COMPETITION, 1991 63

5. Find the maximum value of


Z y q
x4 + (y − y 2 )2 dx
0

for 0 ≤ y ≤ 1.

6. Let A(n) denote the number of sums of positive integers

a1 + a 2 + · · · + a r

which add up to n with

a1 > a 2 + a 3 , a2 > a 3 + a 4 , . . .

ar−2 > ar−1 + ar , ar−1 > ar


Let B(n) denote the number of b1 + b2 + · · · + bs which add up to n, with

(a) b1 ≥ b2 ≥ . . . ≥ bs ,
(b) each bi is in the sequence 1, 2, 4, . . . , gj , . . . defined by g1 = 1, g2 = 2, and
gj = gj−1 + gj−2 + 1, and
(c) if b1 = gk then every element in {1, 2, 4, . . . , gk } appears at least once as a bi .

Prove that A(n) = B(n) for each n ≥ 1.


(For example, A(7) = 5 because the relevant sums are 7, 6 + 1, 5 + 2, 4 + 3, 4 + 2 + 1,
and B(7) = 5 because the relevant sums are 4 + 2 + 1, 2 + 2 + 2 + 1, 2 + 2 + 1 + 1 +
1, 2 + 1 + 1 + 1 + 1 + 1, 1 + 1 + 1 + 1 + 1 + 1 + 1.)

7. For each integer n ≥ 0, let S(n) = n − m2 , where m is the greatest integer with
m2 ≤ n. Define a sequence (ak )∞ k=0 by a0 = A and ak+1 = ak + S(ak ) for k ≥ 0. For
what positive integers A is this sequence eventually constant?

8. Suppose f and g are non-constant, differentiable, real-valued functions defined on


(−∞, ∞). Furthermore, suppose that for each pair of real numbers x and y,

f (x + y) = f (x)f (y) − g(x)g(y),


g(x + y) = f (x)g(y) + g(x)f (y).

If f 0 (0) = 0, prove that (f (x))2 + (g(x))2 = 1 for all x.

9. Does there exist a real number L such that, if m and n are integers greater than L,
then an m × n rectangle may be expressed as a nion of 4 × 6 and 5 × 7 rectangles,
any two of which intersect at most along their boundaries?
64 CHAPTER 2. WILLIAM LOWELL PUTNAM COMPETITION

10. Suppose p is an odd prime. Prove that


p ! !
X p p+j
≡ 2p + 1 (mod p2 ).
j=0 j j

11. Let p be an odd prime and let Zp denote (the field of) integers modulo p. How many
elements are in the set

{x2 : x ∈ Zp } ∩ {y 2 + 1 : y ∈ Zp }?

12. Let a and b be positive numbers. Find the largest number c, in terms of a and b, such
that
sinh ux sinh u(1 − x)
ax b1−x ≤ a +b
sinh u sinh u
for all u with 0 < |u| ≤ c and for all x, 0 < x < 1. (Note: sinh u = (eu − e−u )/2.)

2.8 53th Anual William Lowell Putnam Competition,


1992
1. Prove that f (n) = 1 − n is the only integer-valued function defined on the integers
that satisfies the following conditions.

(i) f (f (n)) = n, for all integers n;


(ii) f (f (n + 2) + 2) = n for all integers n;
(iii) f (0) = 1.

2. Define C(α) to be the coefficient of x1992 in the power series about x = 0 of (1 + x)α .
Evaluate !
Z 1 1992
X 1
C(−y − 1) dy.
0 k=1 y + k

3. For a given positive integer m, find all triples (n, x, y) of positive integers, with n
relatively prime to m, which satisfy

(x2 + y 2 )m = (xy)n .
2.8. 53T H ANUAL WILLIAM LOWELL PUTNAM COMPETITION, 1992 65

4. Let f be an infinitely differentiable real-valued function defined on the real numbers.


If
n2
 
1
f = 2 , n = 1, 2, 3, . . . ,
n n +1
compute the values of the derivatives f (k) (0), k = 1, 2, 3, . . ..

5. For each positive integer n, let an = 0 (or 1) if the number of 1’s in the binary
representation of n is even (or odd), respectively. Show that there do not exist
positive integers k and m such that

ak+j = ak+m+j = ak+m+2j ,

for 0 ≤ j ≤ m − 1.

6. Four points are chosen at random on the surface of a sphere. What is the probability
that the center of the sphere lies inside the tetrahedron whose vertices are at the
four points? (It is understood that each point is independently chosen relative to a
uniform distribution on the sphere.)

7. Let S be a set of n distinct real numbers. Let AS be the set of numbers that occur
as averages of two distinct elements of S. For a given n ≥ 2, what is the smallest
possible number of elements in AS ?

8. For nonnegative integers n and k, define Q(n, k) to be the coefficient of xk in the


expansion of (1 + x + x2 + x3 )n . Prove that
k
! !
X n n
Q(n, k) = ,
j=0 j k − 2j
 
a
where is the standard binomial coefficient. (Reminder: For integers a and b with
b   
a a! a
a ≥ 0, b
= b!(a−b)!
for 0 ≤ b ≤ a, with b
= 0 otherwise.)

9. For any pair (x, y) of real numbers, a sequence (an (x, y))n≥0 is defined as follows:

a0 (x, y) = x,
(an (x, y))2 + y 2
an+1 (x, y) = , for n ≥ 0.
2
Find the area of the region

{(x, y)|(an (x, y))n≥0 converges}.


66 CHAPTER 2. WILLIAM LOWELL PUTNAM COMPETITION

10. Let p(x) be a nonzero polynomial of degree less than 1992 having no nonconstant
factor in common with x3 − x. Let
!
d1992 p(x) f (x)
=
dx1992 x3 − x g(x)
for polynomials f (x) and g(x). Find the smallest possible degree of f (x).
11. Let Dn denote the value of the (n − 1) × (n − 1) determinant
 
3 1 1 1 ··· 1

 1 4 1 1 ··· 1 

 

 1 1 5 1 ··· 1 

 1 1 1 6 ··· 1 .
 
 .. .. .. .. .. .. 

 . . . . . .


1 1 1 1 ··· n+1
n o
Dn
Is the set n!
bounded?
n≥2

12. Let M be a set of real n × n matrices such that


(i) I ∈ M, where I is the n × n identity matrix;
(ii) if A ∈ M and B ∈ M, then either AB ∈ M or −AB ∈ M, but not both;
(iii) if A ∈ M and B ∈ M, then either AB = BA or AB = −BA;
(iv) if A ∈ M and A 6= I, there is at least one B ∈ M such that AB = −BA.
Prove that M contains at most n2 matrices.

2.9 54th Anual William Lowell Putnam Competition,


1993
1. The horizontal line y = c intersects the curve y = 2x − 3x3 in the first quadrant as
in the figure. Find c so that the areas of the two shaded regions are equal. [Figure
not included. The first region is bounded by the y-axis, the line y = c and the curve;
the other lies under the curve and above the line y = c between their two points of
intersection.]
2. Let (xn )n≥0 be a sequence of nonzero real numbers such that x2n − xn−1 xn+1 = 1 for
n = 1, 2, 3, . . .. Prove there exists a real number a such that xn+1 = axn − xn−1 for
all n ≥ 1.
2.9. 54T H ANUAL WILLIAM LOWELL PUTNAM COMPETITION, 1993 67

3. Let Pn be the set of subsets of {1, 2, . . . , n}. Let c(n, m) be the number of functions
f : Pn → {1, 2, . . . , m} such that f (A ∩ B) = min{f (A), f (B)}. Prove that
m
X
c(n, m) = j n.
j=1

4. Let x1 , x2 , . . . , x19 be positive integers each of which is less than or equal to 93. Let
y1 , y2 , . . . , y93 be positive integers each of which is less than or equal to 19. Prove
that there exists a (nonempty) sum of some xi ’s equal to a sum of some yj ’s.

5. Show that
!2 !2 !2
x2 − x x2 − x x2 − x
Z Z 1 Z 11
−10 11 10
dx + dx + dx
−100 x3 − 3x + 1 1
101
x3 − 3x + 1 101
100
x3 − 3x + 1

is a rational number.

6. The infinite sequence of 2’s and 3’s

2, 3, 3, 2, 3, 3, 3, 2, 3, 3, 3, 2, 3, 3, 2, 3, 3, 3, 2, 3, 3, 3, 2, 3, 3, 3, 2, 3, 3, 2, 3, 3, 3, 2, . . .

as the property that, if one forms a second sequence that records the number of 3’s
between successive 2’s, the result is identical to the iven sequence. Show that there
exists a real number r such that, for ny n, the nth term of the sequence is 2 if and
only if n = 1 + brmc for some nonnegative integer m. (Note: bxrf loor denotes the
largest integer less than or equal to x.)

7. Find the smallest positive integer n such that for every integer m with 0 < m < 1993,
there exists an integer k for which
m k m+1
< < .
1993 n 1994

8. Consider the following game played with a deck of 2n cards numbered from 1 to 2n.
The deck is randomly shuffled and n cards are dealt to each of two players. Beginning
with A, the players take turns discarding one of their remaining cards and announcing
its number. The game ends as soon as the sum of the numbers on the discarded cards
is divisible by 2n + 1. The last person to discard wins the game. Assuming optimal
strategy by both A and B, what is the probability that A wins?

9. Two real numbers x and y are chosen at random in the interval (0,1) with respect
to the uniform distribution. What is the probability that he closest integer to x/y is
even? Express the answer in the form r + sπ, where r and s are rational numbers.
68 CHAPTER 2. WILLIAM LOWELL PUTNAM COMPETITION

10. The function K(x, y) is positive and continuous for 0 ≤ x ≤ 1, 0 ≤ y ≤ 1, and the
functions f (x) and g(x) are positive and continuous for 0 ≤ x ≤ 1. Suppose that for
all x, 0 ≤ x ≤ 1, Z 1
f (y)K(x, y) dy = g(x)
0
and Z 1
g(y)K(x, y) dy = f (x).
0
Show that f (x) = g(x) for 9 ≤ x ≤ 1.
11. Show there do not exist four points in the Euclidean plane such that the pairwise
distances between the points are all odd integers.
12. Let S be a set of three, not necessarily distinct, positive integers. Show that one can
transform S into a set containing 0 by a finite number of applications of the following
rule: Select two of the three integers, say x and y, where x < y and replace them
with 2x and y − x.

2.10 55th Anual William Lowell Putnam Competition,


1994
1. Let (an ) be a sequence of positive reals such that, for all n, an ≤ a2n + a2n+1 . Prove
P
that ∞ n=1 an diverges.

2. Find the positive value of m such that the area in the first quadrant enclosed by the
2
ellipse x9 + y 2 = 1, the x-axis, and the line y = 2x/3 is equal to the area in the first
2
quadrant enclosed by the ellipse x9 + y 2 = 1, the y-axis, and the line y = mx.
3. Prove that the points of an isosceles triangle of side length
√ 1 annot be colored in four
colors such that no two points at distance at least 2 − 2 from each other receive the
same color.
4. Let A and B be 2 × 2 matrices with integer entries such that each of A, A + B, A +
2B, A + 3B, A + 4B has an inverse with integer entries. Prove that the same must
be true of A + 5B.
5. Let (rn ) be a sequence of positive reals with limit 0. Let S be the set of all numbers
expressible in the form ri1 + . . . + ri1994 for positive integers i1 < i2 < . . . < i1994 .
Prove that every interval (a, b) contains a subinterval (c, d) whose intersection with
S is empty.
2.11. 56T H ANUAL WILLIAM LOWELL PUTNAM COMPETITION, 1995 69

6. Let f1 , . . . , f10 be bijections of the integers such that for every integer n, there exists
a sequence i1 , . . . , ik for some k such that fi1 ◦ . . . ◦ fik (0) = n. Prove that if A is
any nonempty finite set, there exist at most 512 sequences (e1 , . . . , e10 ) of zeroes and
ones such that f1e1 ◦ . . . ◦ f10e10
maps A to A. (Here f 1 = f and f 0 means the identity
function.)
7. Find all positive integers n such that |n − m2 | ≤ 250 for exactly 15 nonnegative
integers m.
8. Find all c such that the graph of the function x4 + 9x3 + cx2 + ax + b meets some line
in four distinct points.
9. Let f (x) be a positive-valued function over the reals such that f 0 (x) > f (x) for all x.
For what k must there exist N such that f (x) > ekx for x > N ?
!
3 2
10. Let A be the matrix and for positive integers n, define dn as the greatest
4 2
common divisor of the entries of An − I, where I = ((10)(01)). Prove that dn → ∞
as n → ∞.
11. Fix n a positive integer. For α real, define fα (i) as the greatest integer less than or
equal to αi, and write f k for the k-th iterate of f (i.e. f 1 = f and f k+1 = f ◦ f k ).
Prove there exists α such that fαk (n2 ) = fαk (n2 ) = n2 − k for k = 1, . . . , n.
12. Suppose a, b, c, d are integers with 0 ≤ a ≤ bleq99, 0 ≤ c ≤ d ≤ 99. For any integer i,
let ni = 101i + 1002i . Show that if na + nb is congruent to nc + nd mod 10100, then
a = c and b = d.

2.11 56th Anual William Lowell Putnam Competition,


1995
1. Let S be a set of real numbers which is closed under multiplication (that is, if a and
b are in S, then so is ab). Let T and U be disjoint subsets of S whose union is S.
Given that the product of any three (not necessarily distinct) elements of T is in T
and that the product of any three elements of U is in U , show that at least one of
the two subsets T, U is closed under multiplication.
2. For what pairs (a, b) of positive real numbers does the improper integral
Z ∞ q√
√ q√ √ 
x+a− x− x− x − b dx
b
70 CHAPTER 2. WILLIAM LOWELL PUTNAM COMPETITION

converge?

3. The number d1 d2 . . . d9 has nine (not necessarily distinct) decimal digits. The number
e1 e2 . . . e9 is such that each of the nine 9-digit numbers formed by replacing just one
of the digits di is d1 d2 . . . d9 by the corresponding digit ei (1 ≤ i ≤ 9) is divisible by 7.
The number f1 f2 . . . f9 is related to e1 e2 . . . e9 is the same way: that is, each of the nine
numbers formed by replacing one of the ei by the corresponding fi is divisible by 7.
Show that, for each i, di −fi is divisible by 7. [For example, if d1 d2 . . . d9 = 199501996,
then e6 may be 2 or 9, since 199502996 and 199509996 are multiples of 7.]

4. Suppose we have a necklace of n beads. Each bead is labeled with an integer and the
sum of all these labels is n − 1. Prove that we can cut the necklace to form a string
whose consecutive labels x1 , x2 , . . . , xn satisfy
k
X
xi ≤ k − 1 for k = 1, 2, . . . , n.
i=1

5. Let x1 , x2 , . . . , xn be differentiable (real-valued) functions of a single variable f which


satisfy

dx1
= a11 x1 + a12 x2 + · · · + a1n xn
dt
dx2
= a21 x1 + a22 x2 + · · · + a2n xn
dt
.. ..
. .
dxn
= an1 x1 + an2 x2 + · · · + ann xn
dt
for some constants aij > 0. Suppose that for all i, xi (t) → 0 as t → ∞. Are the
functions x1 , x2 , . . . , xn necessarily linearly dependent?

6. Suppose that each of n people writes down the numbers 1,2,3 in random order in
one column of a 3 × n matrix, with all orders equally likely and with the orders for
different columns independent of each other. Let the row sums a, b, c of the resulting
matrix be rearranged (if necessary) so that a ≤ b ≤ c. Show that for some n ≥ 1995,
it is at least four times as likely that both b = a + 1 and c = a + 2 as that a = b = c.

7. For a partition π of {1, 2, 3, 4, 5, 6, 7, 8, 9}, let π(x) be the number of elements in the
part containing x. Prove that for any two partitions π and π 0 , there are two distinct
numbers x and y in {1, 2, 3, 4, 5, 6, 7, 8, 9} such that π(x) = π(y) and π 0 (x) = π 0 (y).
[A partition of a set S is a collection of disjoint subsets (parts) whose union is S.]
2.12. 57T H ANUAL WILLIAM LOWELL PUTNAM COMPETITION, 1996 71

8. An ellipse,
 whose
 semi-axes have lengths a and b, rolls without slipping on the curve
x
y = c sin a . How are a, b, c related, given that the ellipse completes one revolution
when it traverses one period of the curve?
9. To each positive integer with n2 decimal digits, we associate the determinant of the
matrix obtained by writing the digits in order across
! the rows. For example, for
8 6
n = 2, to the integer 8617 we associate det = 50. Find, as a function of n,
1 7
the sum of all the determinants associated with n2 -digit integers. (Leading digits are
assumed to be nonzero; for example, for n = 2, there are 9000 determinants.)
10. Evaluate v
u
u 1
8
t 2207 − 1 .
2207 − 2207−...

a+b c
Express your answer in the form d
, where a, b, c, d are integers.
11. A game starts with four heaps of beans, containing 3,4,5 and 6 beans. The two players
move alternately. A move consists of taking either
a) one bean from a heap, provided at least two beans are left behind in that heap,
or
b) a complete heap of two or three beans.
The player who takes the last heap wins. To win the game, do you want to move first
or second? Give a winning strategy.
12. For a positive real number α, define
S(α) = {bnαc : n = 1, 2, 3, . . .}.
Prove that {1, 2, 3, . . .} cannot be expressed as the disjoint union of three sets S(α), S(β)
and S(γ). [As usual, bxc is the greatest integer ≤ x.]

2.12 57th Anual William Lowell Putnam Competition,


1996
1. Find the least number A such that for any two squares of combined area 1, a rectangle
of area A exists such that the two squares can be packed in the rectangle (without
interior overlap). You may assume that the sides of the squares are parallel to the
sides of the rectangle.
72 CHAPTER 2. WILLIAM LOWELL PUTNAM COMPETITION

2. Let C1 and C2 be circles whose centers are 10 units apart, and whose radii are 1 and
3. Find, with proof, the locus of all points M for which there exists points X on C1
and Y on C2 such that M is the midpoint of the line segment XY .

3. Suppose that each of 20 students has made a choice of anywhere from 0 to 6 courses
from a total of 6 courses offered. Prove or disprove: there are 5 students and 2 courses
such that all 5 have chosen both courses or all 5 have chosen neither course.

4. Let S be the set of ordered triples (a, b, c) of distinct elements of a finite set A.
Suppose that

(a) (a, b, c) ∈ S if and only if (b, c, a) ∈ S;


(b) (a, b, c) ∈ S if and only if (c, b, a) ∈
/ S;
(c) (a, b, c) and (c, d, a) are both in S if and only if (b, c, d) and (d, a, b) are both in
S.

Prove that there exists a one-to-one function g from A to R such that g(a) < g(b) <
g(c) implies (a, b, c) ∈ S. Note: R is the set of real numbers.

5. If p is a prime number greater than 3 and k = b2p/3c, prove that the sum
! ! !
p p p
+ +···+
1 2 k

of binomial coefficients is divisible by p2 .


6. Let c > 0 be a constant. Give a complete description, with proof, of the set of all
continuous functions f : R → R such that f (x) = f (x2 + c) for all x ∈ R. Note that
R denotes the set of real numbers.

7. Define a selfish set to be a set which has its own cardinality (number of elements)
as an element. Find, with proof, the number of subsets of {1, 2, . . . , n} which are
minimal selfish sets, that is, selfish sets none of whose proper subsets is selfish.

8. Show that for every positive integer n,


  2n−1   2n+1
2n − 1 2 2n + 1 2
< 1 · 3 · 5 · · · (2n − 1) < .
e e

9. Given that {x1 , x2 , . . . , xn } = {1, 2, . . . , n}, find, with proof, the largest possible value,
as a function of n (with n ≥ 2), of

x1 x2 + x2 x3 + · · · + xn−1 xn + xn x1 .
2.13. 58T H ANUAL WILLIAM LOWELL PUTNAM COMPETITION, 1997 73

10. For any square matrix A, we can define sin A by the usual power series:

X (−1)n
sin A = A2n+1 .
n=0 (2n + 1)!

Prove or disprove: there exists a 2 × 2 matrix A with real entries such that
!
1 1996
sin A = .
0 1

11. Given a finite string S of symbols X and O, we write ∆(S) for the number of X’s in
S minus the number of O’s. For example, ∆(XOOXOOX) = −1. We call a string
S balanced if every substring T of (consecutive symbols of) S has −2 ≤ ∆(T ) ≤ 2.
Thus, XOOXOOX is not balanced, since it contains the substring OOXOO. Find,
with proof, the number of balanced strings of length n.

12. Let (a1 , b1 ), (a2 , b2 ), . . . , (an , bn ) be the vertices of a convex polygon which contains
the origin in its interior. Prove that there exist positive real numbers x and y such
that
(a1 , b1 )xa1 y b1 + (a2 , b2 )xa2 y b2 + · · · + (an , bn )xan y bn = (0, 0)

2.13 58th Anual William Lowell Putnam Competition,


1997
1. A rectangle, HOM F , has sides HO = 11 and OM = 5. A triangle ABC has H
as the intersection of the altitudes, O the center of the circumscribed circle, M the
midpoint of BC, and F the foot of the altitude from A. What is the length of BC?

2. Players 1, 2, 3, . . . , n are seated around a table, and each has a single penny. Player
1 passes a penny to player 2, who then passes two pennies to player 3. Player 3 then
passes one penny to Player 4, who passes two pennies to Player 5, and so on, players
alternately passing one penny or two to the next player who still has some pennies.
A player who runs out of pennies drops out of the game and leaves the table. Find
an infinite set of numbers n for which some player ends up with all n pennies.

3. Evaluate
! !
x3 x5 x7 x2 x4 x6
Z ∞
x− + − +··· 1 + 2 + 2 2 + 2 2 2 + · · · dx.
0 2 2·4 2·4·6 2 2 ·4 2 ·4 ·6
74 CHAPTER 2. WILLIAM LOWELL PUTNAM COMPETITION

4. Let G be a group with identity e and φ : G → G a function such that


φ(g1 )φ(g2 )φ(g3 ) = φ(h1 )φ(h2 )φ(h3 )
whenever g1 g2 g3 = e = h1 h2 h3 . Prove that there exists an element a ∈ G such that
ψ(x) = aφ(x) is a homomorphism (i.e. ψ(xy) = ψ(x)ψ(y) for all x, y ∈ G).
5. Let Nn denote the number of ordered n-tuples of positive integers (a1 , a2 , . . . , an ) such
that 1/a1 + 1/a2 + . . . + 1/an = 1. Determine whether N10 is even or odd.
6. For a positive integer n and any real number c, define xk recursively by x0 = 0,
x1 = 1, and for k ≥ 0,
cxk+1 − (n − k)xk
xk+2 = .
k+1
Fix n and then take c to be the largest value for which xn+1 = 0. Find xk in terms
of n and k, 1 ≤ k ≤ n.
7. Let {x} denote the distance between the real number x and the nearest integer. For
each positive integer n, evaluate
6n−1
X m m
Fn = min({ }, { }).
m=1 6n 3n
(Here min(a, b) denotes the minimum of a and b.)
8. Let f be a twice-differentiable real-valued function satisfying
f (x) + f 00 (x) = −xg(x)f 0 (x),
where g(x) ≥ 0 for all real x. Prove that |f (x)| is bounded.
P
9. For each positive integer n, write the sum nm=1 1/m in the form pn /qn , where pn and
qn are relatively prime positive integers. Determine all n such that 5 does not divide
qn .
10. Let am,n denote the coefficient of xn in the expansion of (1 + x + x2 )m . Prove that
for all [integers] k ≥ 0,
b 2k
3
c
X
0≤ (−1)i ak−i,i ≤ 1.
i=0

11. Prove that for n ≥ 2,


n terms n − 1 terms
z }| { z }| {
2 ···2 ···2
2 ≡ 22 (mod n).
2.14. 59T H ANUAL WILLIAM LOWELL PUTNAM COMPETITION, 1998 75

12. The dissection of the 3–4–5 triangle shown below (into four congruent right triangles
similar to the original) has diameter 5/2. Find the least diameter of a dissection of
this triangle into four parts. (The diameter of a dissection is the least upper bound
of the distances between pairs of points belonging to the same part.)

2.14 59th Anual William Lowell Putnam Competition,


1998
1. A right circular cone has base of radius 1 and height 3. A cube is inscribed in the
cone so that one face of the cube is contained in the base of the cone. What is the
side-length of the cube?
2. Let s be any arc of the unit circle lying entirely in the first quadrant. Let A be the
area of the region lying below s and above the x-axis and let B be the area of the
region lying to the right of the y-axis and to the left of s. Prove that A + B depends
only on the arc length, and not on the position, of s.
3. Let f be a real function on the real line with continuous third derivative. Prove that
there exists a point a such that
f (a) · f 0 (a) · f 00 (a) · f 000 (a) ≥ 0.

4. Let A1 = 0 and A2 = 1. For n > 2, the number An is defined by concatenating the


decimal expansions of An−1 and An−2 from left to right. For example A3 = A2 A1 = 10,
A4 = A3 A2 = 101, A5 = A4 A3 = 10110, and so forth. Determine all n such that 11
divides An .
5. Let F be a finite collection of open discs in R2 whose union contains a set E ⊆ R2 .
Show that there is a pairwise disjoint subcollection D1 , . . . , Dn in F such that
E ⊆ ∪nj=1 3Dj .
Here, if D is the disc of radius r and center P , then 3D is the disc of radius 3r and
center P .
6. Let A, B, C denote distinct points with integer coordinates in R2 . Prove that if
(|AB| + |BC|)2 < 8 · [ABC] + 1
then A, B, C are three vertices of a square. Here |XY | is the length of segment XY
and [ABC] is the area of triangle ABC.
76 CHAPTER 2. WILLIAM LOWELL PUTNAM COMPETITION

7. Find the minimum value of


(x + 1/x)6 − (x6 + 1/x6 ) − 2
(x + 1/x)3 + (x3 + 1/x3 )

for x > 0.

8. Given a point (a, b) with 0 < b < a, determine the minimum perimeter of a triangle
with one vertex at (a, b), one on the x-axis, and one on the line y = x. You may
assume that a triangle of minimum perimeter exists.

9. let H be the unit hemisphere {(x, y, z) : x2 + y 2 + z 2 = 1, z ≥ 0}, C the unit circle


{(x, y, 0) : x2 + y 2 = 1}, and P the regular pentagon inscribed in C. Determine the
surface area of that portion of H lying over the planar region inside P , and write
your answer in the form A sin α + B cos β, where A, B, α, β are real numbers.

10. Find necessary and sufficient conditions on positive integers m and n so that
mn−1
X
(−1)bi/mc+bi/nc = 0.
i=0

11. Let N be the positive integer with 1998 decimal digits, all of them 1; that is,

N = 1111 · · · 11.

Find the thousandth digit after the decimal point of N.

12. Prove that, for any integers a, b, c, there exists a positive integer n such that n3 + an2 + bn + c
is not an integer.

2.15 60th Anual William Lowell Putnam Competition,


1999
1. Find polynomials f (x),g(x), and h(x), if they exist, such that for all x,


−1 if x < −1
|f (x)| − |g(x)| + h(x) = 3x + 2 if −1 ≤ x ≤ 0


−2x + 2 if x > 0.
2.15. 60T H ANUAL WILLIAM LOWELL PUTNAM COMPETITION, 1999 77

2. Let p(x) be a polynomial that is nonnegative for all real x. Prove that for some k,
there are polynomials f1 (x), . . . , fk (x) such that
k
X
p(x) = (fj (x))2 .
j=1

3. Consider the power series expansion



1 X
= a n xn .
1 − 2x − x2 n=0

Prove that, for each integer n ≥ 0, there is an integer m such that

a2n + a2n+1 = am .

4. Sum the series ∞ X



X m2 n
m m n
.
m=1 n=1 3 (n3 + m3 )

5. Prove that there is a constant C such that, if p(x) is a polynomial of degree 1999,
then Z 1
|p(0)| ≤ C |p(x)| dx.
−1

6. The sequence (an )n≥1 is defined by a1 = 1, a2 = 2, a3 = 24, and, for n ≥ 4,

6a2n−1 an−3 − 8an−1 a2n−2


an = .
an−2 an−3
Show that, for all n, an is an integer multiple of n.

7. Right triangle ABC has right angle at C and ∠BAC = θ; the point D is chosen on
AB so that |AC| = |AD| = 1; the point E is chosen on BC so that ∠CDE = θ. The
perpendicular to BC at E meets AB at F . Evaluate limθ→0 |EF |.

8. Let P (x) be a polynomial of degree n such that P (x) = Q(x)P 00 (x), where Q(x) is a
quadratic polynomial and P 00 (x) is the second derivative of P (x). Show that if P (x)
has at least two distinct roots then it must have n distinct roots.

9. Let A = {(x, y) : 0 ≤ x, y < 1}. For (x, y) ∈ A, let


X
S(x, y) = xm y n ,
1
2
≤m
n
≤2
78 CHAPTER 2. WILLIAM LOWELL PUTNAM COMPETITION

where the sum ranges over all pairs (m, n) of positive integers satisfying the indicated
inequalities. Evaluate

lim (1 − xy 2 )(1 − x2 y)S(x, y).


(x,y)→(1,1),(x,y)∈A

10. Let f be a real function with a continuous third derivative such that f (x), f 0 (x), f 00 (x), f 000 (x)
are positive for all x. Suppose that f 000 (x) ≤ f (x) for all x. Show that f 0 (x) < 2f (x)
for all x.

11. For an integer n ≥ 3, let θ = 2π/n. Evaluate the determinant of the n × n matrix
I +A, where I is the n×n identity matrix and A = (ajk ) has entries ajk = cos(jθ+kθ)
for all j, k.

12. Let S be a finite set of integers, each greater than 1. Suppose that for each integer
n there is some s ∈ S such that gcd(s, n) = 1 or gcd(s, n) = s. Show that there exist
s, t ∈ S such that gcd(s, t) is prime.
2.16. 61ST ANUAL WILLIAM LOWELL PUTNAM COMPETITION, 2000 79

2.16 61st Anual William Lowell Putnam Competition,


2000
P∞
1. Let A be a positive real number. What are the possible values of j=0 x2j , given that
P
x0 , x1 , . . . are positive numbers for which ∞j=0 xj = A?

2. Prove that there exist infinitely many integers n such that n, n + 1, n + 2 are each the
sum of the squares of two integers. [Example: 0 = 02 + 02 , 1 = 02 + 12 , 2 = 12 + 12 .]

3. The octagon P1 P2 P3 P4 P5 P6 P7 P8 is inscribed in a circle, with the vertices around the


circumference in the given order. Given that the polygon P1 P3 P5 P7 is a square of
area 5, and the polygon P2 P4 P6 P8 is aectangle of area 4, find the maximum possible
area of the octagon.
4. Show that the improper integral
Z B
lim sin(x) sin(x2 ) dx
B→∞ 0

converges.

5. Three distinct points with integer coordinates lie in the plane on a circle of radius
r > 0. Show that two of these points are separated by a distance of at least r 1/3 .

6. Let f (x) be a polynomial with integer coefficients. Define a sequence a0 , a1 , . . . of


integers such that a0 = 0 and an+1 = f (an ) for all n ≥ 0. Prove that if there exists a
positive integer m for which am = 0 then either a1 = 0 or a2 = 0.

7. Let aj , bj , cj be integers for 1 ≤ j ≤ N . Assume for each j, at least one of aj , bj , cj is


odd. Show that there exist integers r, s, t such that raj + sbj + tcj is odd for at least
4N/7 values of j, 1 ≤ j ≤ N .

8. Prove that the expression !


gcd(m, n) n
n m
is an integer for all pairs of integers n ≥ m ≥ 1.
P
9. Let f (t) = Nj=1 aj sin(2πjt), where each aj is real and aN is not equal to 0. Let Nk
k
denote the number of zeroes (including multiplicities) of ddtkf . Prove that

N0 ≤ N1 ≤ N2 ≤ · · · and lim Nk = 2N.


k→∞

[Editorial clarification: only zeroes in [0, 1) should be counted.]


80 CHAPTER 2. WILLIAM LOWELL PUTNAM COMPETITION

10. Let f (x) be a continuous function such that f (2x2 − 1) = 2xf (x) for all x. Show that
f (x) = 0 for −1 ≤ x ≤ 1.

11. Let S0 be a finite set of positive integers. We define finite sets S1 , S2 , . . . of positive
integers as follows: the integer a is in Sn+1 if and only if exactly one of a − 1 or a is
in Sn . Show that there exist infinitely many integers N for which SN = S0 ∪ {N + a :
a ∈ S0 }.
n+1
12. Let B be a set of more than 2 n distinct points with coordinates of the form
(±1, ±1, . . . , ±1) in n-dimensional space with n ≥ 3. Show that there are three
distinct points in B which are the vertices of an equilateral triangle.

2.17 62nd Anual William Lowell Putnam Competition,


2001
1. Consider a set S and a binary operation ∗, i.e., for each a, b ∈ S, a ∗ b ∈ S. Assume
(a ∗ b) ∗ a = b for all a, b ∈ S. Prove that a ∗ (b ∗ a) = b for all a, b ∈ S.

2. You have coins C1 , C2 , . . . , Cn . For each k, Ck is biased so that, when tossed, it has
probability 1/(2k+1) of falling heads. If the n coins are tossed, what is the probability
that the number of heads is odd? Express the answer as a rational function of n.

3. For each integer m, consider the polynomial

Pm (x) = x4 − (2m + 4)x2 + (m − 2)2 .

For what values of m is Pm (x) the product of two non-constant polynomials with
integer coefficients?

4. Triangle ABC has an area 1. Points E, F, G lie, respectively, on sides BC, CA, AB
such that AE bisects BF at point R, BF bisects CG at point S, and CG bisects AE
at point T . Find the area of the triangle RST .

5. Prove that there are unique positive integers a, n such that an+1 − (a + 1)n = 2001.

6. Can an arc of a parabola inside a circle of radius 1 have a length greater than 4?

7. Let n be an even positive integer. Write the numbers 1, 2, . . . , n2 in the squares of an


n × n grid so that the k-th row, from left to right, is

(k − 1)n + 1, (k − 1)n + 2, . . . , (k − 1)n + n.


2.18. 63RD ANUAL WILLIAM LOWELL PUTNAM COMPETITION, 2002 81

Color the squares of the grid so that half of the squares in each row and in each column
are red and the other half are black (a checkerboard coloring is one possibility). Prove
that for each coloring, the sum of the numbers on the red squares is equal to the sum
of the numbers on the black squares.

8. Find all pairs of real numbers (x, y) satisfying the system of equations
1 1
x
+ 2y
= (x2 + 3y 2 )(3x2 + y 2 )
1 1
x
− 2y
= 2(y 4 − x4 ).

9. For any positive integer n, let hni denote the closest integer to n. Evaluate

X 2hni + 2−hni
.
n=1 2n

10. Let S denote the set of rational numbers different from {−1, 0, 1}. Define f : S → S
by f (x) = x − 1/x. Prove or disprove that

\
f (n) (S) = ∅,
n=1

where f (n) denotes f composed with itself n times.

11. Let a and b be real numbers in the interval (0, 1/2), and let g be a continuous real-
valued function such that g(g(x)) = ag(x) + bx for all real x. Prove that g(x) = cx
for some constant c.

12. Assume that (an )n≥1 is an increasing sequence of positive real numbers such that
lim an /n = 0. Must there exist infinitely many positive integers n such that an−i +
an+i < 2an for i = 1, 2, . . . , n − 1?

2.18 63rd Anual William Lowell Putnam Competition,


2002
1 Pn (x)
1. Let k be a fixed positive integer. The n-th derivative of xk −1
has the form (xk −1)n+1
where Pn (x) is a polynomial. Find Pn (1).

2. Given any five points on a sphere, show that some four of them must lie on a closed
hemisphere.
82 CHAPTER 2. WILLIAM LOWELL PUTNAM COMPETITION

3. Let n ≥ 2 be an integer and Tn be the number of non-empty subsets S of {1, 2, 3, . . . , n}


with the property that the average of the elements of S is an integer. Prove that Tn −n
is always even.

4. In Determinant Tic-Tac-Toe, Player 1 enters a 1 in an empty 3 × 3 matrix. Player


0 counters with a 0 in a vacant position, and play continues in turn until the 3 × 3
matrix is completed with five 1’s and four 0’s. Player 0 wins if the determinant is 0
and player 1 wins otherwise. Assuming both players pursue optimal strategies, who
will win and how?

5. Define a sequence by a0 = 1, together with the rules a2n+1 = an and a2n+2 = an +an+1
for each integer n ≥ 0. Prove that every positive rational number appears in the set
   
an−1 1 1 2 1 3
:n≥1 = , , , , ,... .
an 1 2 1 3 2

6. Fix an integer b ≥ 2. Let f (1) = 1, f (2) = 2, and for each n ≥ 3, define f (n) = nf (d),
where d is the number of base-b digits of n. For which values of b does

X 1
n=1 f (n)

converge?

7. Shanille O’Keal shoots free throws on a basketball court. She hits the first and misses
the second, and thereafter the probability that she hits the next shot is equal to the
proportion of shots she has hit so far. What is the probability she hits exactly 50 of
her first 100 shots?

8. Consider a polyhedron with at least five faces such that exactly three edges emerge
from each of its vertices. Two players play the following game:

Each player, in turn, signs his or her name on a previously unsigned


face. The winner is the player who first succeeds in signing three faces
that share a common vertex.

Show that the player who signs first will always win by playing as well as possible.

9. Show that, for all integers n > 1,


 n
1 1 1 1
< − 1− < .
2ne e n ne
2.18. 63RD ANUAL WILLIAM LOWELL PUTNAM COMPETITION, 2002 83

10. An integer n, unknown to you, has been randomly chosen in the interval [1, 2002]
with uniform probability. Your objective is to select n in an odd number of guesses.
After each incorrect guess, you are informed whether n is higher or lower, and you
must guess an integer on your next turn among the numbers that are still feasibly
correct. Show that you have a strategy so that the chance of winning is greater than
2/3.

11. A palindrome in base b is a positive integer whose base-b digits read the same back-
wards and forwards; for example, 2002 is a 4-digit palindrome in base 10. Note that
200 is not a palindrome in base 10, but it is the 3-digit palindrome 242 in base 9, and
404 in base 7. Prove that there is an integer which is a 3-digit palindrome in base b
for at least 2002 different values of b.

12. Let p be a prime number. Prove that the determinant of the matrix
 
x y z
 p p
 x y zp  
p 2 p 2 p2
x y z

is congruent modulo p to a product of polynomials of the form ax+by+cz, where a, b, c


are integers. (We say two integer polynomials are congruent modulo p if corresponding
coefficients are congruent modulo p.)
Chapter 3

Asiatic Pacific Mathematical


Olympiads

3.1 1st Asiatic Pacific Mathematical Olympiad, 1989


1. Let x1 , x2 , . . . , xn be positive real numbers, and let
S = x1 + x2 + · · · + x n .
Prove that
S2 S3 Sn
(1 + x1 )(1 + x2 ) · · · (1 + xn ) ≤ 1 + S + + +···+ .
2! 3! n!
2. Prove that the equation
6(6a2 + 3b2 + c2 ) = 5n2
has no solutions in integers except a = b = c = n = 0.
3. Let A1 , A2 , A3 be three points in the plane, and for convenience, let A4 = A1 ,
A5 = A2 . For n = 1, 2, and 3, suppose that Bn is the midpoint of An An+1 , and
suppose that Cn is the midpoint of An Bn . Suppose that An Cn+1 and Bn An+2 meet
at Dn , and that An Bn+1 and Cn An+2 meet at En . Calculate the ratio of the area of
triangle 4D1 D2 D3 to the area of triangle 4E1 E2 E3 .
4. Let S be a set consisting of m pairs (a, b) of positive integers with the property that
1 ≤ a < b ≤ n. Show that there are at least
2
(m − n4 )
4m ·
3n

84
3.2. 2N D ASIATIC PACIFIC MATHEMATICAL OLYMPIAD, 1990 85

triples (a, b, c) such that (a, b), (a, c), and (b, c) belong to S.

5. Determine all functions f from the reals to the reals for which
(1) f (x) is strictly increasing,
(2) f (x) + g(x) = 2x for all real x,
where g(x) is the composition inverse function to f (x). (Note: f and g are said to
be composition inverses if f (g(x)) = x and g(f (x)) = x for all real x.)

3.2 2nd Asiatic Pacific Mathematical Olympiad, 1990


1. Given triagnle ABC, let D, E, F be the midpoints of BC, AC, AB respectively and
let G be the centroid of the triangle.
For each value of ∠BAC, how many non-similar triangles are there in which AEGF
is a cyclic quadrilateral?

2. Let a1 , a2 , . . . , an be positive real numbers, and let Sk be the sum of the products of
a1 , a2 , . . . , an taken k at a time. Show that
!2
n
Sk Sn−k ≥ a1 a2 · · · an
k

for k = 1, 2, . . . , n − 1.

3. Consider all the triangles ABC which have a fixed base AB and whose altitude from
C is a constant h. For which of these triangles is the product of its altitudes a
maximum?

4. A set of 1990 persons is divided into non-intersecting subsets in such a way that:

(a) No one in a subset knows all the others in the subset,


(b) Among any three persons in a subset, there are always at least two who do not
know each other, and
(c) For any two persons in a subset who do not know each other, there is exactly
one person in the same subset knowing both of them.

a Prove that within each subset, every person has the same number of acquain-
tances.
b Determine the maximum possible number of subsets.
86 CHAPTER 3. ASIATIC PACIFIC MATHEMATICAL OLYMPIADS

Note: It is understood that if a person A knows person B, then person B will know
person A; an acquaintance is someone who is known. Every person is assumed to
know one’s self.
5. Show that for every integer n ≥ 6, there exists a convex hexagon which can be
dissected into exactly n congruent triangles.

3.3 3rd Asiatic Pacific Mathematical Olympiad, 1991


1. Let G be the centroid of triangle 4ABC and M be the midpoint of BC. Let X be
on AB and Y on AC such that the points X, Y , and G are collinear and XY and
BC are parallel. Suppose that XC and GB intersect at Q and Y B and GC intersect
at P . Show that triangle 4M P Q is similar to triangle 4ABC.
2. Suppose there are 997 points given in a plane. If every two points are joined by a
line segment with its midpoint coloured in red, show that there are at least 1991 red
points in the plane. Can you find a special case with exactly 1991 red points?
3. Let a1 , a2 , . . . , an , b1 , b2 , . . . , bn be positive real numbers such that a1 +a2 +· · ·+an =
b1 + b2 + · · · + bn . Show that
a21 a22 a2n a1 + a 2 + · · · + a n
+ +···+ ≥
a1 + b 1 a2 + b 2 an + b n 2
4. During a break, n children at school sit in a circle around their teacher to play a
game. The teacher walks clockwise close to the children and hands out candies to
some of them according to the following rule. He selects one child and gives him a
candy, then he skips the next child and gives a candy to the next one, then he skips 2
and gives a candy to the next one, then he skips 3, and so on. Determine the values
of n for which eventually, perhaps after many rounds, all children will have at least
one candy each.
5. Given are two tangent circles and a point P on their common tangent perpendicular
to the lines joining their centres. Construct with ruler and compass all the circles
that are tangent to these two circles and pass through the point P .

3.4 4th Asiatic Pacific Mathematical Olympiad, 1992


1. A triangle with sides a, b, and c is given. Denote by s the semiperimeter, that is
3.5. 5T H ASIATIC PACIFIC MATHEMATICAL OLYMPIAD, 1993 87

s = a+b+c
2
. Construct a triangle with sides s − a, s − b, and s − c. This process is
repeated until a triangle can no longer be constructed with the side lengths given.
For which original triangles can this process be repeated indefinitely?

2. In a circle C with centre O and radius r, let C1 , C2 be two circles with centres O1 ,
O2 and radii r1 , r2 respectively, so that each circle Ci is internally tangent to C at
Ai and so that C1 , C2 are externally tangent to each other at A.Prove that the three
lines OA, O1 A2 , and O2 A1 are concurrent.

3. Let n be an integer such that n > 3. Suppose that we choose three numbers from the
set {1, 2, . . . , n}. Using each of these three numbers only once and using addition,
multiplication, and parenthesis, let us form all possible combinations.

(a) Show that if we choose all three numbers greater than n/2, then thealues of
these combinations are all distinct.

(b) Let p be a prime number such that p ≤ n. Show that the number of ways
of choosing three numbers so that the smallest one is p and the values of the
combinations are not all distinct is precisely the number of positive divisors of
p − 1.

4. Determine all pairs (h, s) of positive integers with the following property: If one draws
h horizontal lines and another s lines which satisfy:

i they are not horizontal,


ii no two of them are parallel,
iii no three of the h + s lines are concurrent,

then the number of regions formed by these h + s lines is 1992.

5. Find a sequence of maximal length consisting of non-zero integers in which the sum
of any seven consecutive terms is positive and that of any eleven consecutive terms
is negative.

3.5 5th Asiatic Pacific Mathematical Olympiad, 1993


1. Let ABCD be a quadrilateral such that all sides have equal length and angle ^ABC is
60 deg. Let l be a line passing through D and not intersecting the quadrilateral (except
at D). Let E and F be the points of intersection of l with AB and BC respectively.
Let M be the point of intersection of CE and AF . Prove that CA2 = CM · CE.
88 CHAPTER 3. ASIATIC PACIFIC MATHEMATICAL OLYMPIADS

2. Find the total number of different integer values the function


5x
f (x) = [x] + [2x] + [ ] + [3x] + [4x]
3
takes for real numbers x with 0 ≤ x ≤ 100.

3. Let (
f (x) = an xn + an−1 xn−1 + · · · + a0 and
g(x) = cn+1 xn+1 + cn xn + · · · + c0
be non-zero polynomials with real coefficients such that g(x) = (x + r)f (x) for some
real number r. If a = max(|an |, . . . , |a0 |) and c = max(|cn+1 |, . . . , |c0 |), prove that
a
c
≤ n + 1.

4. Determine all positive integers n for which the equation

xn + (2 + x)n + (2 − x)n = 0

has an integer as a solution.

5. Let P1 , P2 , . . . , P1993 = P0 be distinct points in the xy-plane with the following


properties:

i both coordinates of Pi are integers, for i = 1, 2, . . . , 1993;


ii there is no point other than Pi and Pi+1 on the line segment joining Pi with Pi+1
whose coordinates are both integers, for i = 0, 1, . . . , 1992.

Prove that for some i, 0 ≤ i ≤ 1992, there exists a point Q with coordinates (qx , qy )
on the line segment joining Pi with Pi+1 such that both 2qx and 2qy are odd integers.

3.6 6th Asiatic Pacific Mathematical Olympiad, 1994


1. Let f : R → R be a function such that:

i For all x, y ∈ R,

f (x) + f (y) + 1 ≥ f (x + y) ≥ f (x) + f (y)

ii For all x ∈ [0, 1), f (0) ≥ f (x),


iii −f (−1) = f (1) = 1.
3.7. 7T H ASIATIC PACIFIC MATHEMATICAL OLYMPIAD, 1995 89

Find all such functions f .

2. Given a nondegenerate triangle 4ABC, with circumcentre O, orthocentre H, and


circumradius R, prove that |OH| < 3R.
3. Let n be an integer of the form a2√+ b2 , where a and b are relatively prime integers
and such that if p is a prime, p ≤ n, then p divides ab. Determine all such n.
4. Is there an infinite set of points in the plane such that no three points are collinear,
and the distance between any two points is rational?
5. You are given three lists A, B, and C. List A contains the numbers of the form 10k
in base 10, with k any integer greater than or equal to 1. Lists B and C contain the
same numbers translated into base 2 and 5 respectively:
A B C
10 1010 20
100 1100100 400
1000 1111101000 13000
.. .. ..
. . .
Prove that for every integer n > 1, there is exactly one number in exactly one of the
lists B or C that has exactly n digits.

3.7 7th Asiatic Pacific Mathematical Olympiad, 1995


1. Determine all sequences of real numbers a1 , a2 , . . . , a1995 which satisfy:
q
2 an − (n − 1) ≥ an+1 − (n − 1), for n = 1, 2, . . . 1994,

and √
2 a1995 − 1994 ≥ a1 + 1.

2. Let a1 , a2 , . . . , an be a sequence of integers with values between 2 and 1995 such


that:
i Any two of the ai ’s are realtively prime,
ii Each ai is either a prime or a product of primes.
Determine the smallest possible values of n to make sure that the sequence will contain
a prime number.
90 CHAPTER 3. ASIATIC PACIFIC MATHEMATICAL OLYMPIADS

3. Let P QRS be a cyclic quadrilateral such that the segments P Q and RS are not
parallel. Consider the set of circles through P and Q, and the set of circles through
R and S. Determine the set A of points of tangency of circles in these two sets.

4. Let C be a circle with radius R and centre O, and S a fixed point in the interior of
C. Let AA0 and BB 0 be perpendicular chords through S. Consider the rectangles
SAM B, SBN 0 A0 , SA0 M 0 B 0 , and SB 0 N A. Find the set of all points M , N 0 , M 0 , and
N when A moves around the whole circle.

5. Find the minimum positive integer k such that there exists a function f from the
set Z of all integers to {1, 2, . . . k} with the property that f (x) 6= f (y) whenever
|x − y| ∈ {5, 7, 12}.

3.8 8th Asiatic Pacific Mathematical Olympiad, 1996


1. Let ABCD be a quadrilateral AB = BC = CD = DA. Let M N and P Q be
two segments perpendicular to the diagonal BD and such that the distance between
them is d > BD/2, with M ∈ AD, N ∈ DC, P ∈ AB, and Q ∈ BC. Show that the
perimeter of hexagon AM N CQP does not depend on the position of M N and P Q
so long as the distance between them remains constant.

2. Let m and n be positive integers such that n ≤ m. Prove that


(m + n)!
2n n! ≤ ≤ (m2 + m)n
(m − n)!

3. Let P1 , P2 , P3 , P4 be four points on a circle, and let I1 be the incentre of the triangle
P2 P3 P4 ; I2 be the incentre of the triangle P1 P3 P4 ; I3 be the incentre of the triangle
P1 P2 P4 ; I4 be the incentre of the triangle P1 P2 P3 . Prove that I1 , I2 , I3 , I4 are the
vertices of a rectangle.

4. The National Marriage Council wishes to invite n couples to form 17 discussion groups
under the following conditions:

(a) All members of a group must be of the same sex; i.e. they are either all male or
all female.
(b) The difference in the size of any two groups is 0 or 1.
(c) All groups have at least 1 member.
(d) Each person must belong to one and only one group.
3.9. 9T H ASIATIC PACIFIC MATHEMATICAL OLYMPIAD, 1997 91

Find all values of n, n ≤ 1996, for which this is possible. Justify your answer.

5. Let a, b, c be the lengths of the sides of a triangle. Prove that


√ √ √ √ √ √
a+b−c+ b+c−a+ c+a−b≤ a+ b+ c ,

and determine when equality occurs.

3.9 9th Asiatic Pacific Mathematical Olympiad, 1997


1. Given
1 1 1
S =1+ 1 + 1 1 +···+ 1 1 1
1+ 3
1+ 3
+ 6
1+ 3
+ +···+
6 1993006

where the denominators contain partial sums of the sequence of reciprocals of trian-
gular numbers (i.e. k = n(n + 1)/2 for n = 1, 2, . . . , 1996). Prove that S > 1001.

2. Find an integer n, where 100 ≤ n ≤ 1997, such that


2n + 2
n
is also an integer.

3. Let 4ABC be a triangle inscribed in a circle and let


ma mb mc
la = , lb = , lc = ,
Ma Mb Mc

where ma , mb , mc are the lengths of the angle bisectors (internal to the triangle) and
Ma , Mb , Mc are the lengths of the angle bisectors extended until they meet the circle.
Prove that
la lb lc
+ + ≥ 3,
sin A sin B sin2 C
2 2

and that equality holds iff ABC is an equilateral triangle.

4. Triangle 4A1 A2 A3 has a right angle at A3 . A sequence of points is now defined by


the following iterative process, where n is a positive integer. From An (n ≥ 3), a
perpendicular line is drawn to meet An−2 An−1 at An+1 .

(a) Prove that if this process is continued indefinitely, then one and only one point
P is interior to every triangle An−2 An−1 An , n ≥ 3.
92 CHAPTER 3. ASIATIC PACIFIC MATHEMATICAL OLYMPIADS

(b) Let A1 and A3 be fixed points. By considering all possible locations of A2 on


the plane, find the locus of P .

5. Suppose that n people A1 , A2 , . . ., An , (n ≥ 3) are seated in a circle and that Ai has


ai objects such that
a1 + a2 + · · · + an = nN,

where N is a positive integer. In order that each person has the same number of
objects, each person Ai is to give or to receive a certain number of objects to or from
its two neighbours Ai−1 and Ai+1 . (Here An+1 means A1 and An means A0 .) How
should this redistribution be performed so that the total number of objects transferred
is minimum?

3.10 10th Asiatic Pacific Mathematical Olympiad, 1998

1. Let F be the set of all n−tuples (A1 , . . . , An ) such that each Ai is a subset of
{1, 2, . . . , 1998}. Let |A| denote the number of elements of the set A. Find:
X
|A1 ∪ · · · ∪ An |
(A1 ,...,An )∈F

2. Show that for any positive integers a and b, (36a + b)(a + 36b) can not be a power of
2.

3. Let a, b, c be positive real numbers. Pruve that:


  !  !
a b c a+b+c
1+ 1+ 1+ ≥2 1+ √
b c a 3
abc

4. Let 4ABC be a triangle and D the foot of the altitude from A. Let E and F lie on
a line through D such that AE is perpendicular to BC, AF is perpendicular to CF ,
and E and F are different from D. Let M and N be the midpoint of the segments
BC and EF , respectively. Prove that AN is perpendicular to N M .

5. √
Find the largest integer n such that n is divisible by all positive integers less than
3
n.
3.11. 11T H ASIATIC PACIFIC MATHEMATICAL OLYMPIAD, 1999 93

3.11 11th Asiatic Pacific Mathematical Olympiad, 1999

1. Find the smallest positive integer n with the following property: there does not exist
an arithmetic progression of 1999 real numbers containing exactly n integers.

2. Let a1 , a2 , . . . be a sequence of real numbers satisfying ai+j ≤ ai + aj for all i, j =


1, 2, . . .. Prove that
a2 a3 an
a1 + + +···+ ≥ an
2 3 n
for each positive integer n.

3. Let Γ1 and Γ2 be two circles intersecting at P and Q. The common tangent, closer
to P , of Γ1 and Γ2 touches Γ1 at A and Γ2 at B. The tangent of Γ1 at P meets Γ2 at
C, which is different from P , and the extension of AP meets BC at R. Prove that
the circumcircle of triangle P QR is tangent to BP and BR.

4. Determine all pairs (a, b) of integers with the property that the numbers a2 + 4b and
b2 + 4a are both perfect squares.

5. Let S be a set of 2n + 1 points in the plane such that no three are collinear and no
four concyclic. A circle will be called good if it has 3 points of S on its circumference,
n − 1 points in its interior and n − 1 points in its exterior. Prove that the number of
good circles has the same parity as n.

3.12 12th Asiatic Pacific Mathematical Olympiad, 2000

1. Compute the sum


101
X x3i
S= 2
i=0 1 − 3xi + 3xi

for
i
xi =
101

2. Given the following triangular arrangement of circles:


94 CHAPTER 3. ASIATIC PACIFIC MATHEMATICAL OLYMPIADS



 

 
 

 
   

   

Each of the numbers 1, . . . , 9 is to be written into one of these circles, so that each
circle contain exactly one of these numbers and:
i the sum of the four numbers on each side of the triangle are equal;
ii the sum of the squares of the four numbers on each side of the triangle are equal.
Find all ways in which this can be done.
3. Let 4ABC be a triangle. Let M and N be the points in which the median and the
angle bisector, respectively, at A meet the side BC. Let Q and P be the point in
which the perpendicular at N to N A meets M A and BA, respectively, and O the
point in which the perpendicular at P to BA meets AN produced. Prove that QO
is perpendicular to BC.
4. Let n, k be given positive integers with n > k. Prove that
1 nn n! nn
· < <
n + 1 k k (n − k)n−k k! (n − k)! k k (n − k)n−k
5. Given a permutation (a0 , a1 , . . . , an ) of the sequence 0, 1, . . . , n. A transposition of
ai with aj is called legal if ai = 0 for i > 0, and ai−1 + 1 = aj . The permu-
tation (a0 , . . . , an ) is called regular if after a number of transpositions it becomes
(1, 2, . . . , n, 0). For which numbers n is the permutation (1, n, n − 1, . . . , 3, 2, 0) reg-
ular?

3.13 13th Asiatic Pacific Mathematical Olympiad, 2001


1. For a positive integer n let S(n) be the sum of digits in the decimal representation of
n. Any positive integer obtained by removing several (at least one) digits from the
3.14. 14T H ASIATIC PACIFIC MATHEMATICAL OLYMPIAD, 2002 95

right-hand end of the decimal representation of n is called a stump of n. Let T (n)


be the sum of all stumps of n. Prove that n = S(n) + 9T (n).

2. Find the largest positive integer N so that the number of integers in the set {1, 2, . . . , N }
which are divisible by 3 is equal to the number of integers which are divisible by 5 or
7 (or both).

3. Let two equal regular n-gons S and T be located in the plane such that their inter-
section is a 2n-gon (n ≥ 3). The sides of the polygon S are coloured in red and the
sides of T in blue. Prove that the sum of the lengths of the blue sides of the polygon
S ∩ T is equal to the sum of the lengths of its red sides.

4. A point in the plane with a cartesian coordinate system is called a mixed point if one
of its coordinates is rational and the other one is irrational. Find all polynomials
with real coefficients such that their graphs do not contain any mixed point.

5. Find the greatest integer n, such that there are n+4 points A, B, C, D, X1 , . . . , Xn in
the plane with AB 6= CD that satisfy the following condition: for each i = 1, 2, . . . , n
triangles ABXi and CDXi are equal.

3.14 14th Asiatic Pacific Mathematical Olympiad, 2002


1. Let a1 , a2 , a3 , . . . , an be a sequence of non-negative integers, where n is a positive
integer. Let
a1 + a 2 + · · · + a n
An = .
n
Prove that
a1 !a2 ! . . . an ! ≥ (bAn c!)n ,
where bAn c is the greatest integer less than or equal to An , and a! = 1 × 2 × · · · × a
for a ≥ 1 (and 0! = 1). When does equality hold?

2. Find all positive integers a and b such that

a2 + b b2 + a
and
b2 − a a2 − b
are both integers.

3. Let 4ABC be an equilateral triangle. Let P be a point on the side AC and Q be a


point on the side AB so that both triangles 4ABP and 4ACQ are acute. Let R be
96 CHAPTER 3. ASIATIC PACIFIC MATHEMATICAL OLYMPIADS

the orthocentre of triangle ABP and S be the orthocentre of triangle ACQ. Let T be
the point common to the segments BP and CQ. Find all possible values of ^CBP
and ^BCQ such that triangle 4T RS is equilateral.
4. Let x, y, z be positive numbers such that
1 1 1
+ + = 1.
x y z
Show that
√ √ √ √ √ √ √
x + yz + y + zx + z + xy ≥ xyz + x + y + z.

5. Let R denote the set of all real numbers. Find all functions f from R to R satisfying:
(i) there are only finitely many s in R such that f (s) = 0, and
(ii) f (x4 + y) = x3 f (x) + f (f (y)) for all x, y in R.

3.15 15th Asiatic Pacific Mathematical Olympiad, 2003


1. Let a, b, c, d, e, f be real numbers such that the polynomial
p(x) = x8 − 4x7 + 7x6 + ax5 + bx4 + cx3 + dx2 + ex + f
factorises into eight linear factors x − xi , with xi > 0 for i = 1, 2, . . . , 8. Determine
all possible values of f .
2. Suppose ABCD is a square piece of cardboard with side length a. On a plane are
two parallel lines `1 and `2 , which are also a units apart. The square ABCD is placed
on the plane so that sides AB and AD intersect `1 at E and F respectively. Also,
sides CB and CD intersect `2 at G and H respectively. Let the perimeters of 4AEF
and 4CGH be m1 and m2 respectively. Prove that no matter how the square was
placed, m1 + m2 remains constant.
3. Let k ≥ 14 be an integer, and let pk be the largest prime number which is strictly
less than k. You may assume that pk ≥ 3k/4. Let n be a composite integer. Prove:
(a) if n = 2pk , then n does not divide (n − k)! ;
(b) if n > 2pk , then n divides (n − k)!
4. Let a, b, c be the sides of a triangle, with a + b + c = 1, and let n ≥ 2 be an integer.
Show that √

n

n
√ n
2
a n + b n + bn + c n + cn + a n < 1 +
n
.
2
3.15. 15T H ASIATIC PACIFIC MATHEMATICAL OLYMPIAD, 2003 97

5. Given two positive integers m and n, find the smallest positive integer k such that
among any k people, either there are 2m of them who form m pairs of mutually
acquainted people or there are 2n of them forming n pairs of mutually unacquainted
people.

You might also like